AH1 EXAM #2

Ace your homework & exams now with Quizwiz!

An emergency department nurse cares for a client who is severely dehydrated and is prescribed 3 L of intravenous fluid over 6 hours. At what rate (mL/hr) should the nurse set the intravenous pump to infuse the fluids? (Record your answer using a whole number.) ____ mL/hr

500 mL/hr Because IV pumps deliver in units of milliliters per hour, the pump would have to be set at 500 mL/hr to deliver 3 L (3000 mL) over 6 hours. 6x = 3000 x = 500

A nurse is caring for a client who weighs 220 pounds and is started on enoxaparin (Lovenox). How much enoxaparin does the nurse anticipate administering? (Record your answer using a whole number.) _____ mg

90 mg The dose of enoxaparin is 1 mg/kg body weight, not to exceed 90 mg. This client weighs 220 pounds (110 kg), and so will get the maximal dose.

A home health nurse is visiting a new client who uses oxygen in the home. For which factors does the nurse assess when determining if the client is using the oxygen safely? (Select all that apply.) a. The client does not allow smoking in the house. b. Electrical cords are in good working order. c. Flammable liquids are stored in the garage. d. Household light bulbs are the fluorescent type. e. The client does not have pets inside the home.

A, B ,C a. The client does not allow smoking in the house. b. Electrical cords are in good working order. c. Flammable liquids are stored in the garage. Oxygen is an accelerant, which means it enhances combustion, so precautions are needed whenever using it. The nurse should assess if the client allows smoking near the oxygen, whether electrical cords are in good shape or are frayed, and if flammable liquids are stored (and used) in the garage away from the oxygen. Light bulbs and pets are not related to oxygen safety.

A hospital nurse is participating in a drill during which many clients with inhalation anthrax are being admitted. What drugs should the nurse anticipate administering? (Select all that apply.) a. Amoxicillin (Amoxil) b. Ciprofloxacin (Cipro) c. Doxycycline (Vibramycin) d. Ethambutol (Myambutol) e. Sulfamethoxazole-trimethoprim (SMX-TMP) (Septra)

A, B, C a. Amoxicillin (Amoxil) b. Ciprofloxacin (Cipro) c. Doxycycline (Vibramycin) Amoxicillin, ciprofloxacin, and doxycycline are all possible treatments for inhalation anthrax. Ethambutol is used for tuberculosis. SMX-TMP is commonly used for urinary tract infections and other common infections.

A nurse is caring for a client with a nonhealing arterial ulcer. The physician has informed the client about possibly needing to amputate the clients leg. The client is crying and upset. What actions by the nurse are best? (Select all that apply.) a. Ask the client to describe his or her current emotions. b. Assess the client for support systems and family. c. Offer to stay with the client if he or she desires. d. Relate how smoking contributed to this situation. e. Tell the client that many people have amputations.

A, B, C a. Ask the client to describe his or her current emotions. b. Assess the client for support systems and family. c. Offer to stay with the client if he or she desires. When a client is upset, the nurse should offer self by remaining with the client if desired. Other helpful measures include determining what and whom the client has for support systems and asking the client to describe what he or she is feeling. Telling the client how smoking has led to this situation will only upset the client further and will damage the therapeutic relationship. Telling the client that many people have amputations belittles the clients feelings.

A nurse teaches a client who has chronic obstructive pulmonary disease. Which statements related to nutrition should the nurse include in this clients teaching? (Select all that apply.) a. Avoid drinking fluids just before and during meals. b. Rest before meals if you have dyspnea. c. Have about six small meals a day. d. Eat high-fiber foods to promote gastric emptying. e. Increase carbohydrate intake for energy.

A, B, C a. Avoid drinking fluids just before and during meals. b. Rest before meals if you have dyspnea. c. Have about six small meals a day. Fluids can make a client feel bloated and should be avoided with meals. Resting before the meal will help a client with dyspnea. Six small meals a day also will help to decrease bloating. Fibrous foods can produce gas, which can cause abdominal bloating and can increase shortness of breath. The client should increase calorie and protein intake to prevent malnourishment. The client should not increase carbohydrate intake as this will increase carbon dioxide production and increase the clients risk of for acidosis.

A nurse teaches a client about self-care after experiencing a urinary calculus treated by lithotripsy. Which statements should the nurse include in this clients discharge teaching? (Select all that apply.) a. Finish the prescribed antibiotic even if you are feeling better. b. Drink at least 3 liters of fluid each day. c. The bruising on your back may take several weeks to resolve. d. Report any blood present in your urine. e. It is normal to experience pain and difficulty urinating.

A, B, C a. Finish the prescribed antibiotic even if you are feeling better. b. Drink at least 3 liters of fluid each day. c. The bruising on your back may take several weeks to resolve. The client should be taught to finish the prescribed antibiotic to ensure that he or she does not get a urinary tract infection. The client should drink at least 3 liters of fluid daily to dilute potential stone-forming crystals, prevent dehydration, and promote urine flow. After lithotripsy, the client should expect bruising that may take several weeks to resolve. The client should also experience blood in the urine for several days. The client should report any pain, fever, chills, or difficulty with urination to the provider as these may signal the beginning of an infection or the formation of another stone.

A nurse assesses a client who is recovering from a heart transplant. Which assessment findings should alert the nurse to the possibility of heart transplant rejection? (Select all that apply.) a. Shortness of breath b. Abdominal bloating c. New-onset bradycardia d. Increased ejection fraction e. Hypertension

A, B, C a. Shortness of breath b. Abdominal bloating c. New-onset bradycardia Clinical manifestations of heart transplant rejection include shortness of breath, fatigue, fluid gain, abdominal bloating, new-onset bradycardia, hypotension, atrial fibrillation or flutter, decreased activity tolerance, and decreased ejection fraction.

A client has been diagnosed with an empyema. What interventions should the nurse anticipate providing to this client? (Select all that apply.) a. Assisting with chest tube insertion b. Facilitating pleural fluid sampling c. Performing frequent respiratory assessment d. Providing antipyretics as needed e. Suctioning deeply every 4 hours

A, B, C, D a. Assisting with chest tube insertion b. Facilitating pleural fluid sampling c. Performing frequent respiratory assessment d. Providing antipyretics as needed The client with an empyema is often treated with chest tube insertion, which facilitates obtaining samples of the pleural fluid for analysis and re-expands the lungs. The nurse should perform frequent respiratory system assessments. Antipyretic medications are also used. Suction is only used when needed and is not done deeply to prevent tissue injury.

A nurse is teaching a client about possible complications and hazards of home oxygen therapy. About which complications does the nurse plan to teach the client? (Select all that apply.) a. Absorptive atelectasis b. Combustion c. Dried mucous membranes d. Oxygen-induced hyperventilation e. Toxicity

A, B, C, E a. Absorptive atelectasis b. Combustion c. Dried mucous membranes e. Toxicity Complications of oxygen therapy include absorptive atelectasis, combustion, dried mucous membranes, and oxygen toxicity. Oxygen-induced hypoventilation is also a complication.

A nursing student planning to teach clients about risk factors for coronary artery disease (CAD) would include which topics? (Select all that apply.) a. Advanced age b. Diabetes c. Ethnic background d. Medication use e. Smoking

A, B, C, E a. Advanced age b. Diabetes c. Ethnic background e. Smoking Age, diabetes, ethnic background, and smoking are all risk factors for developing CAD; medication use is not.

A nurse assesses clients on a cardiac unit. Which clients should the nurse identify as at greatest risk for the development of acute pericarditis? (Select all that apply.) a. A 36-year-old woman with systemic lupus erythematosus (SLE) b. A 42-year-old man recovering from coronary artery bypass graft surgery c. A 59-year-old woman recovering from a hysterectomy d. An 80-year-old man with a bacterial infection of the respiratory tract e. An 88-year-old woman with a stage III sacral ulcer

A, B, D a. A 36-year-old woman with systemic lupus erythematosus (SLE) b. A 42-year-old man recovering from coronary artery bypass graft surgery d. An 80-year-old man with a bacterial infection of the respiratory tract Acute pericarditis is most commonly associated acute exacerbations of systemic connective tissue disease, including SLE; with Dresslers syndrome, or inflammation of the cardiac sac after cardiac surgery or a myocardial infarction; and with infective organisms, including bacterial, viral, and fungal infections. Abdominal and reproductive surgeries and pressure ulcers do not increase clients risk for acute pericarditis.

A client has been bedridden for several days after major abdominal surgery. What action does the nurse delegate to the unlicensed assistive personnel (UAP) for deep vein thrombosis (DVT) prevention? (Select all that apply.) a. Apply compression stockings. b. Assist with ambulation. c. Encourage coughing and deep breathing. d. Offer fluids frequently. e. Teach leg exercises.

A, B, D a. Apply compression stockings. b. Assist with ambulation. d. Offer fluids frequently. The UAP can apply compression stockings, assist with ambulation, and offer fluids frequently to help prevent DVT. The UAP can also encourage the client to do pulmonary exercises, but these do not decrease the risk of DVT. Teaching is a nursing function.

A nurse prepares to discharge a client who has heart failure. Which questions should the nurse ask to ensure this clients safety prior to discharging home? (Select all that apply.) a. Are your bedroom and bathroom on the first floor? b. What social support do you have at home? c. Will you be able to afford your oxygen therapy?d. What spiritual beliefs may impact your recovery? e. Are you able to accurately weigh yourself at home?

A, B, D a. Are your bedroom and bathroom on the first floor? b. What social support do you have at home? d. What spiritual beliefs may impact your recovery? To ensure safety upon discharge, the nurse should assess for structural barriers to functional ability, such as stairs. The nurse should also assess the clients available social support, which may include family, friends, and home health services. The clients ability to adhere to medication and treatments, including daily weights, should also be reviewed. The other questions do not address the clients safety upon discharge.

A nurse is caring for a client on IV infusion of heparin. What actions does this nurse include in the clients plan of care? (Select all that apply.) a. Assess the client for bleeding. b. Monitor the daily activated partial thromboplastin time (aPTT) results. c. Stop the IV for aPTT above baseline. d. Use an IV pump for the infusion. e. Weigh the client daily on the same scale.

A, B, D a. Assess the client for bleeding. b. Monitor the daily activated partial thromboplastin time (aPTT) results. d. Use an IV pump for the infusion. Assessing for bleeding, monitoring aPTT, and using an IV pump for the infusion are all important safety measures for heparin to prevent injury from bleeding. The aPTT needs to be 1.5 to 2 times normal in order to demonstrate that the heparin is therapeutic. Weighing the client is not related.

A nurse assesses a client who has had two episodes of bacterial cystitis in the last 6 months. Which questions should the nurse ask? (Select all that apply.) a. How much water do you drink every day? b. Do you take estrogen replacement therapy? c. Does anyone in your family have a history of cystitis? d. Are you on steroids or other immune-suppressing drugs? e. Do you drink grapefruit juice or orange juice daily?

A, B, D a. How much water do you drink every day? b. Do you take estrogen replacement therapy? d. Are you on steroids or other immune-suppressing drugs? Fluid intake, estrogen levels, and immune suppression all can increase the chance of recurrent cystitis. Family history is usually insignificant, and cranberry juice, not grapefruit or orange juice, has been found to increase the acidic pH and reduce the risk for bacterial cystitis.

A nurse assesses a client with nephrotic syndrome. For which clinical manifestations should the nurse assess? (Select all that apply.) a. Proteinuria b. Hypoalbuminemia c. Dehydration d. Lipiduria e. Dysuria f. Costovertebral angle (CVA) tenderness

A, B, D a. Proteinuria b. Hypoalbuminemia d. Lipiduria Nephrotic syndrome is caused by glomerular damage and is characterized by proteinuria (protein level higher than 3.5 g/24 hr), hypoalbuminemia, edema, and lipiduria. Fluid overload leading to edema and hypertension is common with nephrotic syndrome; dehydration does not occur. Dysuria is present with cystitis. CVA tenderness is present with inflammatory changes in the kidney.

A nurse cares for clients with urinary incontinence. Which types of incontinence are correctly paired with their clinical manifestation? (Select all that apply.) a. Stress incontinence Urine loss with physical exertion b. Urge incontinence Large amount of urine with each occurrence c. Functional incontinence Urine loss results from abnormal detrusor contractions d. Overflow incontinence Constant dribbling of urine e. Reflex incontinence Leakage of urine without lower urinary tract disorder

A, B, D a. Stress incontinence Urine loss with physical exertion b. Urge incontinence Large amount of urine with each occurrence d. Overflow incontinence Constant dribbling of urine Stress incontinence is a loss of urine with physical exertion, coughing, sneezing, or exercising. Urge incontinence presents with an abrupt and strong urge to void and usually has a large amount of urine released with each occurrence. Overflow incontinence occurs with bladder distention and results in a constant dribbling of urine. Functional incontinence is the leakage of urine caused by factors other than a disorder of the lower urinary tract. Reflex incontinence results from abnormal detrusor contractions from a neurologic abnormality.

A nurse teaches a client with polycystic kidney disease (PKD). Which statements should the nurse include in this clients discharge teaching? (Select all that apply.) a. Take your blood pressure every morning. b. Weigh yourself at the same time each day. c. Adjust your diet to prevent diarrhea. d. Contact your provider if you have visual disturbances. e. Assess your urine for renal stones.

A, B, D a. Take your blood pressure every morning. b. Weigh yourself at the same time each day. d. Contact your provider if you have visual disturbances. A client who has PKD should measure and record his or her blood pressure and weight daily, limit salt intake, and adjust dietary selections to prevent constipation. The client should notify the provider if urine smells foul or has blood in it, as these are signs of a urinary tract infection or glomerular injury. The client should also notify the provider if visual disturbances are experienced, as this is a sign of a possible berry aneurysm, which is a complication of PKD. Diarrhea and renal stones are not manifestations or complications of PKD; therefore, teaching related to these concepts would be inappropriate.

A nurse prepares to discharge a client who has heart failure. Based on the Heart Failure Core Measure Set, which actions should the nurse complete prior to discharging this client? (Select all that apply.) a. Teach the client about dietary restrictions. b. Ensure the client is prescribed an angiotensin-converting enzyme (ACE) inhibitor. c. Encourage the client to take a baby aspirin each day. d. Confirm that an echocardiogram has been completed. e. Consult a social worker for additional resources.

A, B, D a. Teach the client about dietary restrictions. b. Ensure the client is prescribed an angiotensin-converting enzyme (ACE) inhibitor. d. Confirm that an echocardiogram has been completed. The Heart Failure Core Measure Set includes discharge instructions on diet, activity, medications, weight monitoring and plan for worsening symptoms, evaluation of left ventricular systolic function (usually with an echocardiogram), and prescribing an ACE inhibitor or angiotensin receptor blocker. Aspirin is not part of the Heart Failure Core Measure Set and is usually prescribed for clients who experience a myocardial infarction. Although the nurse may consult the social worker or case manager for additional resources, this is not part of the Core Measures.

A nursing student studying acute coronary syndromes learns that the pain of a myocardial infarction (MI) differs from stable angina in what ways? (Select all that apply.) a. Accompanied by shortness of breath b. Feelings of fear or anxiety c. Lasts less than 15 minutes d. No relief from taking nitroglycerin e. Pain occurs without known cause

A, B, D, E a. Accompanied by shortness of breath b. Feelings of fear or anxiety d. No relief from taking nitroglycerin e. Pain occurs without known cause The pain from an MI is often accompanied by shortness of breath and fear or anxiety. It lasts longer than 15 minutes and is not relieved by nitroglycerin. It occurs without a known cause such as exertion.

A nurse is planning discharge teaching on tracheostomy care for an older client. What factors does the nurse need to assess before teaching this particular client? (Select all that apply.) a. Cognition b. Dexterity c. Hydration d. Range of motion e. Vision

A, B, D, E a. Cognition b. Dexterity d. Range of motion e. Vision The older adult is at risk for having impairments in cognition, dexterity, range of motion, and vision that could limit the ability to perform tracheostomy care and should be assessed. Hydration is not directly related to the ability to perform self-care.

A nurse teaches a client with a history of calcium phosphate urinary stones. Which statements should the nurse include in this clients dietary teaching? (Select all that apply.) a. Limit your intake of food high in animal protein. b. Read food labels to help minimize your sodium intake. c. Avoid spinach, black tea, and rhubarb. d. Drink white wine or beer instead of red wine. e. Reduce your intake of milk and other dairy products.

A, B, E a. Limit your intake of food high in animal protein. b. Read food labels to help minimize your sodium intake. e. Reduce your intake of milk and other dairy products. Clients with calcium phosphate urinary stones should be taught to limit the intake of foods high in animal protein, sodium, and calcium. Clients with calcium oxalate stones should avoid spinach, black tea, and rhubarb. Clients with uric acid stones should avoid red wine.

A nurse is assessing a client with left-sided heart failure. For which clinical manifestations should the nurse assess? (Select all that apply.) a. Pulmonary crackles b. Confusion, restlessness c. Pulmonary hypertension d. Dependent edema e. Cough that worsens at night

A, B, E a. Pulmonary crackles b. Confusion, restlessness e. Cough that worsens at night Left-sided heart failure occurs with a decrease in contractility of the heart or an increase in afterload. Most of the signs will be noted in the respiratory system. Right-sided heart failure occurs with problems from the pulmonary vasculature onward including pulmonary hypertension. Signs will be noted before the right atrium or ventricle including dependent edema.

A nurse teaches a female client who has stress incontinence. Which statements should the nurse include about pelvic muscle exercises? (Select all that apply.) a. When you start and stop your urine stream, you are using your pelvic muscles. b. Tighten your pelvic muscles for a slow count of 10 and then relax for a slow count of 10. c. Pelvic muscle exercises should only be performed sitting upright with your feet on the floor. d. After you have been doing these exercises for a couple days, your control of urine will improve. e. Like any other muscle in your body, you can make your pelvic muscles stronger by contracting them.

A, B, E a. When you start and stop your urine stream, you are using your pelvic muscles. b. Tighten your pelvic muscles for a slow count of 10 and then relax for a slow count of 10. e. Like any other muscle in your body, you can make your pelvic muscles stronger by contracting them. The client should be taught that the muscles used to start and stop urination are pelvic muscles, and that pelvic muscles can be strengthened by contracting and relaxing them. The client should tighten pelvic muscles for a slow count of 10 and then relax the muscles for a slow count of 10, and perform this exercise 15 times while in lying-down, sitting-up, and standing positions. The client should begin to notice improvement in control of urine after several weeks of exercising the pelvic muscles.

A nurse evaluates laboratory results for a client with heart failure. Which results should the nurse expect? (Select all that apply.) a. Hematocrit: 32.8% b. Serum sodium: 130 mEq/L c. Serum potassium: 4.0 mEq/L d. Serum creatinine: 1.0 mg/dL e. Proteinuria f. Microalbuminuria

A, B, E, F a. Hematocrit: 32.8% b. Serum sodium: 130 mEq/L e. Proteinuria f. Microalbuminuria A hematocrit of 32.8% is low (should be 42.6%), indicating a dilutional ratio of red blood cells to fluid. A serum sodium of 130 mEq/L is low because of hemodilution. Microalbuminuria and proteinuria are present, indicating a decrease in renal filtration. These are early warning signs of decreased compliance of the heart. The potassium level is on the high side of normal and the serum creatinine level is normal.

A nurse collaborates with an unlicensed assistive personnel (UAP) to provide care for a client with congestive heart failure. Which instructions should the nurse provide to the UAP when delegating care for this client? (Select all that apply.) a. Reposition the client every 2 hours. b. Teach the client to perform deep-breathing exercises. c. Accurately record intake and output. d. Use the same scale to weigh the client each morning. e. Place the client on oxygen if the client becomes short of breath.

A, C, D a. Reposition the client every 2 hours. c. Accurately record intake and output. d. Use the same scale to weigh the client each morning. The UAP should reposition the client every 2 hours to improve oxygenation and prevent atelectasis. The UAP can also accurately record intake and output, and use the same scale to weigh the client each morning before breakfast. UAPs are not qualified to teach clients or assess the need for and provide oxygen therapy.

A nurse is providing pneumonia vaccinations in a community setting. Due to limited finances, the event organizers must limit giving the vaccination to priority groups. What clients would be considered a priority when administering the pneumonia vaccination? (Select all that apply.) a. 22-year-old client with asthma b. Client who had a cholecystectomy last year c. Client with well-controlled diabetes d. Healthy 72-year-old client e. Client who is taking medication for hypertension

A, C, D, E a. 22-year-old client with asthma c. Client with well-controlled diabetes d. Healthy 72-year-old client e. Client who is taking medication for hypertension Clients over 65 years of age and any client (no matter what age) with a chronic health condition would be considered a priority for a pneumonia vaccination. Having a cholecystectomy a year ago does not qualify as a chronic health condition.

A client is being discharged on warfarin (Coumadin) therapy. What discharge instructions is the nurse required to provide? (Select all that apply.) a. Dietary restrictions b. Driving restrictions c. Follow-up laboratory monitoring d. Possible drug-drug interactions e. Reason to take medication

A, C, D, E a. Dietary restrictions c. Follow-up laboratory monitoring d. Possible drug-drug interactions e. Reason to take medication The Joint Commissions Core Measures state that clients being discharged on warfarin need instruction on follow-up monitoring, dietary restrictions, drug-drug interactions, and reason for compliance. Driving is typically not restricted.

A nurse is caring for a client who had coronary artery bypass grafting yesterday. What actions does the nurse delegate to the unlicensed assistive personnel (UAP)? (Select all that apply.) a. Assist the client to the chair for meals and to the bathroom. b. Encourage the client to use the spirometer every 4 hours. c. Ensure the client wears TED hose or sequential compression devices. d. Have the client rate pain on a 0-to-10 scale and report to the nurse. e. Take and record a full set of vital signs per hospital protocol.

A, C, E a. Assist the client to the chair for meals and to the bathroom. c. Ensure the client wears TED hose or sequential compression devices. e. Take and record a full set of vital signs per hospital protocol. The nurse can delegate assisting the client to get up in the chair or ambulate to the bathroom, applying TEDs or sequential compression devices, and taking/recording vital signs. The spirometer should be used every hour the day after surgery. Assessing pain using a 0-to-10 scale is a nursing assessment, although if the client reports pain, the UAP should inform the nurse so a more detailed assessment is done.

A nurse cares for a client who is prescribed an intravenous prostacyclin agent. Which actions should the nurse take to ensure the clients safety while on this medication? (Select all that apply.) a. Keep an intravenous line dedicated strictly to the infusion. b. Teach the client that this medication increases pulmonary pressures. c. Ensure that there is always a backup drug cassette available. d. Start a large-bore peripheral intravenous line. e. Use strict aseptic technique when using the drug delivery system.

A, C, E a. Keep an intravenous line dedicated strictly to the infusion. c. Ensure that there is always a backup drug cassette available. e. Use strict aseptic technique when using the drug delivery system. Intravenous prostacyclin agents should be administered in a central venous catheter with a dedicated intravenous line for this medication. Death has been reported when the drug delivery system is interrupted; therefore, a backup drug cassette should also be available. The nurse should use strict aseptic technique when using the drug delivery system. The nurse should teach the client that this medication decreases pulmonary pressures and increases lung blood flow.

A nurse teaches a client about self-catheterization in the home setting. Which statements should the nurse include in this clients teaching? (Select all that apply.) a. Wash your hands before and after self-catheterization. b. Use a large-lumen catheter for each catheterization. c. Use lubricant on the tip of the catheter before insertion. d. Self-catheterize at least twice a day or every 12 hours. e. Use sterile gloves and sterile technique for the procedure. f. Maintain a specific schedule for catheterization.

A, C, F a. Wash your hands before and after self-catheterization. c. Use lubricant on the tip of the catheter before insertion. f. Maintain a specific schedule for catheterization. The key points in self-catheterization include washing hands, using lubricants, and maintaining a regular schedule to avoid distention and retention of urine that leads to bacterial growth. A smaller rather than a larger lumen catheter is preferred. The client needs to catheterize more often than every 12 hours. Self-catheterization in the home is a clean procedure.

A nurse is caring for a client who has a tracheostomy tube. What actions may the nurse delegate to unlicensed assistive personnel (UAP)? (Select all that apply.) a. Applying water-soluble lip balm to the clients lips b. Ensuring the humidification provided is adequate c. Performing oral care with alcohol-based mouthwash d. Reminding the client to cough and deep breathe often e. Suctioning excess secretions through the tracheostomy

A, D a. Applying water-soluble lip balm to the clients lips d. Reminding the client to cough and deep breathe often The UAP can perform hygiene measures such as applying lip balm and reinforce teaching such as reminding the client to perform coughing and deep-breathing exercises. Oral care can be accomplished with normal saline, not products that dry the mouth. Ensuring the humidity is adequate and suctioning through the tracheostomy are nursing functions.

The nurse working in the emergency department knows that which factors are commonly related to aneurysm formation? (Select all that apply.) a. Atherosclerosis b. Down syndrome c. Frequent heartburn d. History of hypertension e. History of smoking

A, D, E a. Atherosclerosis d. History of hypertension e. History of smoking Atherosclerosis, hypertension, hyperlipidemia, and smoking are the most common related factors. Down syndrome and heartburn have no relation to aneurysm formation.

A client is being discharged home after having a tracheostomy placed. What suggestions does the nurse offer to help the client maintain self-esteem? (Select all that apply.) a. Create a communication system. b. Dont go out in public alone. c. Find hobbies to enjoy at home. d. Try loose-fitting shirts with collars. e. Wear fashionable scarves.

A, D, E a. Create a communication system. d. Try loose-fitting shirts with collars. e. Wear fashionable scarves. The client with a tracheostomy may be shy and hesitant to go out in public. The client should have a sound communication method to ease frustration. The nurse can also suggest ways of enhancing appearance so the client is willing to leave the house. These can include wearing scarves and loose-fitting shirts to hide the stoma. Keeping the client homebound is not good advice.

After teaching a client with congestive heart failure (CHF), the nurse assesses the clients understanding. Which client statements indicate a correct understanding of the teaching related to nutritional intake? (Select all that apply.) a. Ill read the nutritional labels on food items for salt content. b. I will drink at least 3 liters of water each day. c. Using salt in moderation will reduce the workload of my heart. d. I will eat oatmeal for breakfast instead of ham and eggs. e. Substituting fresh vegetables for canned ones will lower my salt intake.

A, D, E a. Ill read the nutritional labels on food items for salt content. d. I will eat oatmeal for breakfast instead of ham and eggs. e. Substituting fresh vegetables for canned ones will lower my salt intake. Nutritional therapy for a client with CHF is focused on decreasing sodium and water retention to decrease the workload of the heart. The client should be taught to read nutritional labels on all food items, omit table salt and foods high in sodium (e.g., ham and canned foods), and limit water intake to a normal 2 L/day.

A nurse plans care for a client who has chronic obstructive pulmonary disease and thick, tenacious secretions. Which interventions should the nurse include in this clients plan of care? (Select all that apply.) a. Ask the client to drink 2 liters of fluids daily. b. Add humidity to the prescribed oxygen. c. Suction the client every 2 to 3 hours. d. Use a vibrating positive expiratory pressure device. e. Encourage diaphragmatic breathing.

ANS: A, B, D a. Ask the client to drink 2 liters of fluids daily. b. Add humidity to the prescribed oxygen. d. Use a vibrating positive expiratory pressure device. Interventions to decrease thick tenacious secretions include maintaining adequate hydration and providing humidified oxygen. These actions will help to thin secretions, making them easier to remove by coughing. The use of a vibrating positive expiratory pressure device can also help clients remove thick secretions. Although suctioning may assist with the removal of secretions, frequent suctioning can cause airway trauma and does not support the clients ability to successfully remove secretions through normal coughing. Diaphragmatic breathing is not used to improve the removal of thick secretions.

A client in the emergency department is taking rifampin (Rifadin) for tuberculosis. The client reports yellowing of the sclera and skin and bleeding after minor trauma. What laboratory results correlate to this condition? (Select all that apply.) a. Blood urea nitrogen (BUN): 19 mg/dL b. International normalized ratio (INR): 6.3 c. Prothrombin time: 35 seconds d. Serum sodium: 130 mEq/L e. White blood cell (WBC) count: 72,000/mm3

B, C b. International normalized ratio (INR): 6.3 c. Prothrombin time: 35 seconds Rifampin can cause liver damage, evidenced by the clients high INR and prothrombin time. The BUN and WBC count are normal. The sodium level is low, but that is not related to this clients problem.

A nurse assesses a client who has a chest tube. For which manifestations should the nurse immediately intervene? (Select all that apply.) a. Production of pink sputum b. Tracheal deviation c. Sudden onset of shortness of breath d. Pain at insertion sitee. Drainage of 75 mL/hr

B, C b. Tracheal deviation c. Sudden onset of shortness of breath Tracheal deviation and sudden onset of shortness of breath are manifestations of a tension pneumothorax. The nurse must intervene immediately for this emergency situation. Pink sputum is associated with pulmonary edema and is not a complication of a chest tube. Pain at the insertion site and drainage of 75 mL/hr are normal findings with a chest tube.

What nonpharmacologic comfort measures should the nurse include in the plan of care for a client with severe varicose veins? (Select all that apply.) a. Administering mild analgesics for pain b. Applying elastic compression stockings c. Elevating the legs when sitting or lying d. Reminding the client to do leg exercises e. Teaching the client about surgical options

B, C, D b. Applying elastic compression stockings c. Elevating the legs when sitting or lying d. Reminding the client to do leg exercises The three Es of care for varicose veins include elastic compression hose, exercise, and elevation. Mild analgesics are not a nonpharmacologic measure. Teaching about surgical options is not a comfort measure.

A nursing student learns about modifiable risk factors for coronary artery disease. Which factors does this include? (Select all that apply.) a. Age b. Hypertension c. Obesity d. Smoking e. Stress

B, C, D, E b. Hypertension c. Obesity d. Smoking e. Stress Hypertension, obesity, smoking, and excessive stress are all modifiable risk factors for coronary artery disease. Age is a nonmodifiable risk factor.

A nurse assesses a client with chronic obstructive pulmonary disease. Which questions should the nurse ask to determine the clients activity tolerance? (Select all that apply.) a. What color is your sputum? b. Do you have any difficulty sleeping? c. How long does it take to perform your morning routine? d. Do you walk upstairs every day? e. Have you lost any weight lately?

B, C, E b. Do you have any difficulty sleeping? c. How long does it take to perform your morning routine? e. Have you lost any weight lately? Difficulty sleeping could indicate worsening breathlessness, as could taking longer to perform activities of daily living. Weight loss could mean increased dyspnea as the client becomes too fatigued to eat. The color of the clients sputum would not assist in determining activity tolerance. Asking whether the client walks upstairs every day is not as pertinent as determining if the client becomes short of breath on walking upstairs, or if the client goes upstairs less often than previously.

A nurse assesses a client who has a family history of polycystic kidney disease (PKD). For which clinical manifestations should the nurse assess? (Select all that apply.) a. Nocturia b. Flank pain c. Increased abdominal girth d. Dysuria e. Hematuria f. Diarrhea

B, C, E b. Flank pain c. Increased abdominal girth e. Hematuria Clients with PKD experience abdominal distention that manifests as flank pain and increased abdominal girth. Bloody urine is also present with tissue damage secondary to PKD. Clients with PKD often experience constipation, but would not report nocturia or dysuria.

A nurse assesses a client who is diagnosed with infective endocarditis. Which assessment findings should the nurse expect? (Select all that apply.) a. Weight gain b. Night sweats c. Cardiac murmur d. Abdominal bloating e. Oslers nodes

B, C, E b. Night sweats c. Cardiac murmur e. Oslers nodes Clinical manifestations of infective endocarditis include fever with chills, night sweats, malaise and fatigue, anorexia and weight loss, cardiac murmur, and Oslers nodes on palms of the hands and soles of the feet. Abdominal bloating is a manifestation of heart transplantation rejection.

A client with a known abdominal aortic aneurysm reports dizziness and severe abdominal pain. The nurse assesses the clients blood pressure at 82/40 mm Hg. What actions by the nurse are most important? (Select all that apply.) a. Administer pain medication. b. Assess distal pulses every 10 minutes. c. Have the client sign a surgical consent. d. Notify the Rapid Response Team. e. Take vital signs every 10 minutes.

B, D, E b. Assess distal pulses every 10 minutes. d. Notify the Rapid Response Team. e. Take vital signs every 10 minutes. This client may have a ruptured/rupturing aneurysm. The nurse should notify the Rapid Response team and perform frequent client assessments. Giving pain medication will lower the clients blood pressure even further. The nurse cannot have the client sign a consent until the physician has explained the procedure.

A client is 1 day postoperative after a coronary artery bypass graft. What nonpharmacologic comfort measures does the nurse include when caring for this client? (Select all that apply.) a. Administer pain medication before ambulating. b. Assist the client into a position of comfort in bed. c. Encourage high-protein diet selections. d. Provide complementary therapies such as music. e. Remind the client to splint the incision when coughing.

B, D, E b. Assist the client into a position of comfort in bed. d. Provide complementary therapies such as music. e. Remind the client to splint the incision when coughing. Nonpharmacologic comfort measures can include positioning, complementary therapies, and splinting the chest incision. Medications are not nonpharmacologic. Food choices are not comfort measures.

A nurse assesses a client who is recovering from a nephrostomy. Which assessment findings should alert the nurse to urgently contact the health care provider? (Select all that apply.) a. Clear drainage b. Bloody drainage at site c. Client reports headache d. Foul-smelling drainage e. Urine draining from site

B, D, E b. Bloody drainage at site d. Foul-smelling drainage e. Urine draining from site After a nephrostomy, the nurse should assess the client for complications and urgently notify the provider if drainage decreases or stops, drainage is cloudy or foul-smelling, the nephrostomy sites leaks blood or urine, or the client has back pain. Clear drainage is normal. A headache would be an unrelated finding.

A nurse assesses a client who has a mediastinal chest tube. Which symptoms require the nurses immediate intervention? (Select all that apply.) a. Production of pink sputum b. Tracheal deviation c. Pain at insertion site d. Sudden onset of shortness of breath e. Drainage greater than 70 mL/hr f. Disconnection at Y site

B, D, E, F b. Tracheal deviation d. Sudden onset of shortness of breath e. Drainage greater than 70 mL/hr f. Disconnection at Y site Immediate intervention is warranted if the client has tracheal deviation because this could indicate a tension pneumothorax. Sudden shortness of breath could indicate dislodgment of the tube, occlusion of the tube, or pneumothorax. Drainage greater than 70 mL/hr could indicate hemorrhage. Disconnection at the Y site could result in air entering the tubing. Production of pink sputum, oxygen saturation less than 95%, and pain at the insertion site are not signs/symptoms that would require immediate intervention.

A nurse assesses a client with a fungal urinary tract infection (UTI). Which assessments should the nurse complete? (Select all that apply.) a. Palpate the kidneys and bladder. b. Assess the medical history and current medical problems. c. Perform a bladder scan to assess post-void residual. d. Inquire about recent travel to foreign countries. e. Obtain a current list of medications.

B, E b. Assess the medical history and current medical problems. e. Obtain a current list of medications. Clients who are severely immunocompromised or who have diabetes mellitus are more prone to fungal UTIs. The nurse should assess for these factors by asking about medical history, current medical problems, and the current medication list. A physical examination and a post-void residual may be needed, but not until further information is obtained indicating that these examinations are necessary. Travel to foreign countries probably would not be important because, even if exposed, the client needs some degree of compromised immunity to develop a fungal UTI.

A nurse teaches clients about the difference between urge incontinence and stress incontinence. Which statements should the nurse include in this education? (Select all that apply.) a. Urge incontinence involves a post-void residual volume less than 50 mL. b. Stress incontinence occurs due to weak pelvic floor muscles. c. Stress incontinence usually occurs in people with dementia. d. Urge incontinence can be managed by increasing fluid intake. e. Urge incontinence occurs due to abnormal bladder contractions.

B, E b. Stress incontinence occurs due to weak pelvic floor muscles. e. Urge incontinence occurs due to abnormal bladder contractions. Clients who suffer from stress incontinence have weak pelvic floor muscles or urethral sphincter and cannot tighten their urethra sufficiently to overcome the increased detrusor pressure. Stress incontinence is common after childbirth, when the pelvic muscles are stretched and weakened from pregnancy and delivery. Urge incontinence occurs in people who cannot suppress the contraction signal from the detrusor muscle. Abnormal detrusor contractions may be a result of neurologic abnormalities including dementia, or may occur with no known abnormality. Post-void residual is associated with reflex incontinence, not with urge incontinence or stress incontinence. Management of urge incontinence includes decreasing fluid intake, especially in the evening hours.

A nurse reviews laboratory results for a client with glomerulonephritis. The clients glomerular filtration rate (GFR) is 40 mL/min as measured by a 24-hour creatinine clearance. How should the nurse interpret this finding? (Select all that apply.) a. Excessive GFR b. Normal GFR c. Reduced GFR d. Potential for fluid overload e. Potential for dehydration

C, D c. Reduced GFR d. Potential for fluid overload The GFR refers to the initial amount of urine that the kidneys filter from the blood. In the healthy adult, the normal GFR ranges between 100 and 120 mL/min, most of which is reabsorbed in the kidney tubules. A GFR of 40 mL/min is drastically reduced, with the client experiencing fluid retention and risks for hypertension and pulmonary edema as a result of excess vascular fluid.

Which statements by the client indicate good understanding of foot care in peripheral vascular disease? (Select all that apply.) a. A good abrasive pumice stone will keep my feet soft. b. Ill always wear shoes if I can buy cheap flip-flops. c. I will keep my feet dry, especially between the toes. d. Lotion is important to keep my feet smooth and soft. e. Washing my feet in room-temperature water is best.

C, D, E c. I will keep my feet dry, especially between the toes. d. Lotion is important to keep my feet smooth and soft. e. Washing my feet in room-temperature water is best. Good foot care includes appropriate hygiene and injury prevention. Keeping the feet dry; wearing good, comfortable shoes; using lotion; washing the feet in room-temperature water; and cutting the nails straight across are all important measures. Abrasive material such as pumice stones should not be used. Cheap flip- flops may not fit well and wont offer much protection against injury.

A nurse assesses a client with asthma and notes bilateral wheezing, decreased pulse oxygen saturation, and suprasternal retraction on inhalation. Which actions should the nurse take? (Select all that apply.) a. Administer prescribed salmeterol (Serevent) inhaler. b. Assess the client for a tracheal deviation. c. Administer oxygen to keep saturations greater than 94%. d. Perform peak expiratory flow readings. e. Administer prescribed albuterol (Proventil) inhaler.

C, E c. Administer oxygen to keep saturations greater than 94%. e. Administer prescribed albuterol (Proventil) inhaler. Suprasternal retraction caused by inhalation usually indicates that the client is using accessory muscles and is having difficulty moving air into the respiratory passages because of airway narrowing. Wheezing indicates a narrowed airway; a decreased pulse oxygen saturation also supports this finding. The asthma is not responding to the medication, and intervention is needed. Administration of a rescue inhaler is indicated, probably along with administration of oxygen. The nurse would not do a peak flow reading at this time, nor would a code be called. Midline trachea is a normal and expected finding.

A nurse is preparing a client for a femoropopliteal bypass operation. What actions does the nurse delegate to the unlicensed assistive personnel (UAP)? (Select all that apply.) a. Administering preoperative medication b. Ensuring the consent is signed c. Marking pulses with a pen d. Raising the siderails on the bed e. Recording baseline vital signs

D, E d. Raising the siderails on the bed e. Recording baseline vital signs The UAP can raise the siderails of the bed for client safety and take and record the vital signs. Administering medications, ensuring a consent is on the chart, and marking the pulses for later comparison should be done by the registered nurse. This is also often done by the postanesthesia care nurse and is part of the hand-off report.

The nurse instructs a client on the steps needed to obtain a peak expiratory flow rate. In which order should these steps occur? 1. Take as deep a breath as possible. 2. Stand up (unless you have a physical disability). 3. Place the meter in your mouth, and close your lips around the mouthpiece. 4. Make sure the device reads zero or is at base level. 5. Blow out as hard and as fast as possible for 1 to 2 seconds. 6. Write down the value obtained. 7. Repeat the process two additional times, and record the highest number in your chart. a. 4, 2, 1, 3, 5, 6, 7 b. 3, 4, 1, 2, 5, 7, 6 c. 2, 1, 3, 4, 5, 6, 7 d. 1, 3, 2, 5, 6, 7, 4

a. 4, 2, 1, 3, 5, 6, 7 The proper order for obtaining a peak expiratory flow rate is as follows. Make sure the device reads zero or is at base level. The client should stand up (unless he or she has a physical disability). The client should take as deep a breath as possible, place the meter in the mouth, and close the lips around the mouthpiece. The client should blow out as hard and as fast as possible for 1 to 2 seconds. The value obtained should be written down. The process should be repeated two more times, and the highest of the three numbers should be recorded in the clients chart.

A nurse assesses clients on a cardiac unit. Which client should the nurse identify as being at greatest risk for the development of left-sided heart failure?a. A 36-year-old woman with aortic stenosis b. A 42-year-old man with pulmonary hypertension c. A 59-year-old woman who smokes cigarettes daily d. A 70-year-old man who had a cerebral vascular accident

a. A 36-year-old woman with aortic stenosis Although most people with heart failure will have failure that progresses from left to right, it is possible to have left-sided failure alone for a short period. It is also possible to have heart failure that progresses from right to left. Causes of left ventricular failure include mitral or aortic valve disease, coronary artery disease, and hypertension. Pulmonary hypertension and chronic cigarette smoking are risk factors for right ventricular failure. A cerebral vascular accident does not increase the risk of heart failure.

A nurse cares for adult clients who experience urge incontinence. For which client should the nurse plan a habit training program? a. A 78-year-old female who is confused b. A 65-year-old male with diabetes mellitus c. A 52-year-old female with kidney failure d. A 47-year-old male with arthritis

a. A 78-year-old female who is confused For a bladder training program to succeed in a client with urge incontinence, the client must be alert, aware, and able to resist the urge to urinate. Habit training will work best for a confused client. This includes going to the bathroom (or being assisted to the bathroom) at set times. The other clients may benefit from another type of bladder training.

A nurse admits a client from the emergency department. Client data are listed below: History: -70 years of age -History of diabetes -On insulin twice a day -Reports new-onset dyspnea and productive cough Physical Assessment: -Crackles and rhonchi heard throughout the lungs -Dullness to percussion LLL -Afebrile -Oriented to person only Laboratory Values: -WBC: 5,200/mm3 -PaO2 on room air 65 mm Hg What action by the nurse is the priority? a. Administer oxygen at 4 liters per nasal cannula. b. Begin broad-spectrum antibiotics. c. Collect a sputum sample for culture. d. Start an IV of normal saline at 50 mL/hr.

a. Administer oxygen at 4 liters per nasal cannula. All actions are appropriate for this client who has manifestations of pneumonia. However, airway and breathing come first, so begin oxygen administration and titrate it to maintain saturations greater than 95%. Start the IV and collect a sputum culture, and then begin antibiotics.

A nurse is interested in providing community education and screening on hypertension. In order to reach a priority population, to what target audience should the nurse provide this service? a. African-American churches b. Asian-American groceries c. High school sports camps d. Womens health clinics

a. African-American churches African Americans in the United States have one of the highest rates of hypertension in the world. The nurse has the potential to reach this priority population by providing services at African-American churches. Although hypertension education and screening are important for all groups, African Americans are the priority population for this intervention.

A nurse is caring for a client who is intubated and has an intra-aortic balloon pump. The client is restless and agitated. What action should the nurse perform first for comfort? a. Allow family members to remain at the bedside. b. Ask the family if the client would like a fan in the room. c. Keep the television tuned to the clients favorite channel. d. Speak loudly to the client in case of hearing problems.

a. Allow family members to remain at the bedside. Allowing the family to remain at the bedside can help calm the client with familiar voices (and faces if the client wakes up). A fan might be helpful but may also spread germs through air movement. The TV should not be kept on all the time to allow for rest. Speaking loudly may agitate the client more.

The emergency department (ED) manager is reviewing client charts to determine how well the staff performs when treating clients with community-acquired pneumonia. What outcome demonstrates that goals for this client type have been met? a. Antibiotics started before admission b. Blood cultures obtained within 20 minutes c. Chest x-ray obtained within 30 minutes d. Pulse oximetry obtained on all clients

a. Antibiotics started before admission Goals for treatment of community-acquired pneumonia include initiating antibiotics prior to inpatient admission or within 6 hours of presentation to the ED. Timely collection of blood cultures, chest x-ray, and pulse oximetry are important as well but do not coincide with established goals.

A client is receiving oxygen at 4 liters per nasal cannula. What comfort measure may the nurse delegate to unlicensed assistive personnel (UAP)? a. Apply water-soluble ointment to nares and lips. b. Periodically turn the oxygen down or off. c. Remove the tubing from the clients nose. d. Turn the client every 2 hours or as needed.

a. Apply water-soluble ointment to nares and lips. Oxygen can be drying, so the UAP can apply water-soluble lubricant to the clients lips and nares. The UAP should not adjust the oxygen flow rate or remove the tubing. Turning the client is not related to comfort measures for oxygen.

A student is practicing suctioning a tracheostomy in the skills laboratory. What action by the student demonstrates that more teaching is needed? a. Applying suction while inserting the catheter b. Preoxygenating the client prior to suctioning c. Suctioning for a total of three times if needed d. Suctioning for only 10 to 15 seconds each time

a. Applying suction while inserting the catheter Suction should only be applied while withdrawing the catheter. The other actions are appropriate.

A client had a femoropopliteal bypass graft with a synthetic graft. What action by the nurse is most important to prevent wound infection? a. Appropriate hand hygiene before giving care b. Assessing the clients temperature every 4 hours c. Clean technique when changing dressings d. Monitoring the clients daily white blood cell count

a. Appropriate hand hygiene before giving care Hand hygiene is the best way to prevent infections in hospitalized clients. Dressing changes should be done with sterile technique. Assessing vital signs and white blood cell count will not prevent infection.

A nurse is working with a client who takes atorvastatin (Lipitor). The clients recent laboratory results include a blood urea nitrogen (BUN) of 33 mg/dL and creatinine of 2.8 mg/dL. What action by the nurse is best? a. Ask if the client eats grapefruit. b. Assess the client for dehydration. c. Facilitate admission to the hospital. d. Obtain a random urinalysis.

a. Ask if the client eats grapefruit. There is a drug-food interaction between statins and grapefruit that can lead to acute kidney failure. This client has elevated renal laboratory results, indicating some degree of kidney involvement. The nurse should assess if the client eats grapefruit or drinks grapefruit juice. Dehydration can cause the BUN to be elevated, but the elevation in creatinine is more specific for a kidney injury. The client does not necessarily need to be admitted. A urinalysis may or may not be ordered.

A client has been hospitalized with tuberculosis (TB). The clients spouse is fearful of entering the room where the client is in isolation and refuses to visit. What action by the nurse is best? a. Ask the spouse to explain the fear of visiting in further detail. b. Inform the spouse the precautions are meant to keep other clients safe. c. Show the spouse how to follow the isolation precautions to avoid illness. d. Tell the spouse that he or she has already been exposed, so its safe to visit.

a. Ask the spouse to explain the fear of visiting in further detail. The nurse needs to obtain further information about the spouses specific fears so they can be addressed. This will decrease stress and permit visitation, which will be beneficial for both client and spouse. Precautions for TB prevent transmission to all who come into contact with the client. Explaining isolation precautions and what to do when entering the room will be helpful, but this is too narrow in scope to be the best answer. Telling the spouse its safe to visit is demeaning of the spouses feelings.

25. A nurse auscultates a clients lung fields. Which action should the nurse take based on the lung sounds? (Click the media button to hear the audio clip.) a. Assess for airway obstruction. b. Initiate oxygen therapy. c. Assess vital signs. d. Elevate the clients head.

a. Assess for airway obstruction. Stridor is the sound heard, and it indicates severe airway constriction. The nurse must administer a bronchodilator to get air into the lungs. Administering oxygen, assessing vital signs, and elevating the clients head will not help until the clients airways are open.

An older adult is on cardiac monitoring after a myocardial infarction. The client shows frequent dysrhythmias. What action by the nurse is most appropriate? a. Assess for any hemodynamic effects of the rhythm. b. Prepare to administer antidysrhythmic medication. c. Notify the provider or call the Rapid Response Team. d. Turn the alarms off on the cardiac monitor.

a. Assess for any hemodynamic effects of the rhythm. Older clients may have dysrhythmias due to age-related changes in the cardiac conduction system. They may have no significant hemodynamic effects from these changes. The nurse should first assess for the effects of the dysrhythmia before proceeding further. The alarms on a cardiac monitor should never be shut off. The other two actions may or may not be needed.

While assessing a client on a cardiac unit, a nurse identifies the presence of an S3 gallop. Which action should the nurse take next? a. Assess for symptoms of left-sided heart failure. b. Document this as a normal finding. c. Call the health care provider immediately. d. Transfer the client to the intensive care unit.

a. Assess for symptoms of left-sided heart failure. The presence of an S3 gallop is an early diastolic filling sound indicative of increasing left ventricular pressure and left ventricular failure. The other actions are not warranted.

A client is on a dopamine infusion via a peripheral line. What action by the nurse takes priority for safety? a. Assess the IV site hourly. b. Monitor the pedal pulses. c. Monitor the clients vital signs. d. Obtain consent for a central line.

a. Assess the IV site hourly. Dopamine should be infused through a central line to prevent extravasation and necrosis of tissue. If it needs to be run peripherally, the nurse assesses the site hourly for problems. When the client is getting the central line, ensuring informed consent is on the chart is a priority. But at this point, the client has only a peripheral line, so caution must be taken to preserve the integrity of the clients integumentary system. Monitoring pedal pulses and vital signs give indications as to how well the drug is working.

The nurse is assessing a client on admission to the hospital. The clients leg appears as shown below: What action by the nurse is best? a. Assess the clients ankle-brachial index. b. Elevate the clients leg above the heart. c. Obtain an ice pack to provide comfort. d. Prepare to teach about heparin sodium.

a. Assess the clients ankle-brachial index. This client has dependent rubor, a classic finding in peripheral arterial disease. The nurse should measure the clients ankle-brachial index. Elevating the leg above the heart will further impede arterial blood flow. Ice will cause vasoconstriction, also impeding circulation and perhaps causing tissue injury. Heparin sodium is not the drug of choice for this condition.

A client had an inferior wall myocardial infarction (MI). The nurse notes the clients cardiac rhythm as shown below: What action by the nurse is most important? a. Assess the clients blood pressure and level of consciousness. b. Call the health care provider or the Rapid Response Team. c. Obtain a permit for an emergency temporary pacemaker insertion. d. Prepare to administer antidysrhythmic medication.

a. Assess the clients blood pressure and level of consciousness. Clients with an inferior wall MI often have bradycardia and blocks that lead to decreased perfusion, as seen in this ECG strip showing sinus bradycardia. The nurse should first assess the clients hemodynamic status, including vital signs and level of consciousness. The client may or may not need the Rapid Response Team, a temporary pacemaker, or medication; there is no indication of this in the question.

A client with a history of heart failure and hypertension is in the clinic for a follow-up visit. The client is on lisinopril (Prinivil) and warfarin (Coumadin). The client reports new-onset cough. What action by the nurse is most appropriate? a. Assess the clients lung sounds and oxygenation. b. Instruct the client on another antihypertensive. c. Obtain a set of vital signs and document them. d. Remind the client that cough is a side effect of Prinivil.

a. Assess the clients lung sounds and oxygenation. This client could be having an exacerbation of heart failure or be experiencing a side effect of lisinopril (and other angiotensin-converting enzyme inhibitors). The nurse should assess the clients lung sounds and other signs of oxygenation first. The client may or may not need to switch antihypertensive medications. Vital signs and documentation are important, but the nurse should assess the respiratory system first. If the cough turns out to be a side effect, reminding the client is appropriate, but then more action needs to be taken.

An unlicensed assistive personnel (UAP) was feeding a client with a tracheostomy. Later that evening, the UAP reports that the client had a coughing spell during the meal. What action by the nurse takes priority? a. Assess the clients lung sounds. b. Assign a different UAP to the client. c. Report the UAP to the manager. d. Request thicker liquids for meals.

a. Assess the clients lung sounds. The priority is to check the clients oxygenation because he or she may have aspirated. Once the client has been assessed, the nurse can consult with the registered dietitian about appropriately thickened liquids. The UAP should have reported the incident immediately, but addressing that issue is not the immediate priority.

A client has a tracheostomy that is 3 days old. Upon assessment, the nurse notes the clients face is puffy and the eyelids are swollen. What action by the nurse takes priority? a. Assess the clients oxygen saturation. b. Notify the Rapid Response Team. c. Oxygenate the client with a bag-valve-mask. d. Palpate the skin of the upper chest.

a. Assess the clients oxygen saturation. This client may have subcutaneous emphysema, which is air that leaks into the tissues surrounding the tracheostomy. The nurse should first assess the clients oxygen saturation and other indicators of oxygenation. If the client is stable, the nurse can palpate the skin of the upper chest to feel for the air. If the client is unstable, the nurse calls the Rapid Response Team. Using a bag-valve-mask device may or may not be appropriate for the unstable client.

A nurse admits a client who is experiencing an exacerbation of heart failure. Which action should the nurse take first? a. Assess the clients respiratory status. b. Draw blood to assess the clients serum electrolytes. c. Administer intravenous furosemide (Lasix). d. Ask the client about current medications.

a. Assess the clients respiratory status. Assessment of respiratory and oxygenation status is the priority nursing intervention for the prevention of complications. Monitoring electrolytes, administering diuretics, and asking about current medications are important but do not take priority over assessing respiratory status.

A client has been diagnosed with a deep vein thrombosis and is to be discharged on warfarin (Coumadin). The client is adamant about refusing the drug because its dangerous. What action by the nurse is best? a. Assess the reason behind the clients fear. b. Remind the client about laboratory monitoring. c. Tell the client drugs are safer today than before. d. Warn the client about consequences of noncompliance.

a. Assess the reason behind the clients fear. The first step is to assess the reason behind the clients fear, which may be related to the experience of someone the client knows who took warfarin. If the nurse cannot address the specific rationale, teaching will likely be unsuccessful. Laboratory monitoring once every few weeks may not make the client perceive the drug to be safe. General statements like drugs are safer today do not address the root cause of the problem. Warning the client about possible consequences of not taking the drug is not therapeutic and is likely to lead to an adversarial relationship.

A nurse is teaching a client with heart failure who has been prescribed enalapril (Vasotec). Which statement should the nurse include in this clients teaching? a. Avoid using salt substitutes. b. Take your medication with food. c. Avoid using aspirin-containing products. d. Check your pulse daily.

a. Avoid using salt substitutes. Angiotensin-converting enzyme (ACE) inhibitors such as enalapril inhibit the excretion of potassium. Hyperkalemia can be a life-threatening side effect, and clients should be taught to limit potassium intake. Salt substitutes are composed of potassium chloride. ACE inhibitors do not need to be taken with food and have no impact on the clients pulse rate. Aspirin is often prescribed in conjunction with ACE inhibitors and is not contraindicated.

A client admitted for pneumonia has been tachypneic for several days. When the nurse starts an IV to give fluids, the client questions this action, saying I have been drinking tons of water. How am I dehydrated? What response by the nurse is best? a. Breathing so quickly can be dehydrating. b. Everyone with pneumonia is dehydrated. c. This is really just to administer your antibiotics. d. Why do you think you are so dehydrated?

a. Breathing so quickly can be dehydrating. Tachypnea and mouth breathing, both seen in pneumonia, increase insensible water loss and can lead to a degree of dehydration. The other options do not give the client useful information.

A nurse is caring for a client with a nonhealing arterial lower leg ulcer. What action by the nurse is best? a. Consult with the Wound Ostomy Care Nurse. b. Give pain medication prior to dressing changes. c. Maintain sterile technique for dressing changes. d. Prepare the client for eventual amputation.

a. Consult with the Wound Ostomy Care Nurse. A nonhealing wound needs the expertise of the Wound Ostomy Care Nurse (or Wound Ostomy Continence Nurse). Premedicating prior to painful procedures and maintaining sterile technique are helpful, but if the wound is not healing, more needs to be done. The client may need an amputation, but other options need to be tried first.

A nursing student is caring for a client who had a myocardial infarction. The student is confused because the client states nothing is wrong and yet listens attentively while the student provides education on lifestyle changes and healthy menu choices. What response by the faculty member is best? a. Continue to educate the client on possible healthy changes. b. Emphasize complications that can occur with noncompliance. c. Tell the client that denial is normal and will soon go away. d. You need to make sure the client understands this illness.

a. Continue to educate the client on possible healthy changes. Clients are often in denial after a coronary event. The client who seems to be in denial but is compliant with treatment may be using a healthy form of coping that allows time to process the event and start to use problem- focused coping. The student should not discourage this type of denial and coping, but rather continue providing education in a positive manner. Emphasizing complications may make the client defensive and more anxious. Telling the client that denial is normal is placing too much attention on the process. Forcing the client to verbalize understanding of the illness is also potentially threatening to the client.

A nurse is assessing a client with peripheral artery disease (PAD). The client states walking five blocks is possible without pain. What question asked next by the nurse will give the best information? a. Could you walk further than that a few months ago? b. Do you walk mostly uphill, downhill, or on flat surfaces? c. Have you ever considered swimming instead of walking? d. How much pain medication do you take each day?

a. Could you walk further than that a few months ago? As PAD progresses, it takes less oxygen demand to cause pain. Needing to cut down on activity to be pain free indicates the clients disease is worsening. The other questions are useful, but not as important.

A nurse assesses a client who presents with renal calculi. Which question should the nurse ask? a. Do any of your family members have this problem? b. Do you drink any cranberry juice? c. Do you urinate after sexual intercourse? d. Do you experience burning with urination?

a. Do any of your family members have this problem? There is a strong association between family history and stone formation and recurrence. Nephrolithiasis is associated with many genetic variations; therefore, the nurse should ask whether other family members have also had renal stones. The other questions do not refer to renal calculi but instead are questions that should be asked of a client with a urinary tract infection.

A client is in the clinic a month after having a myocardial infarction. The client reports sleeping well since moving into the guest bedroom. What response by the nurse is best? a. Do you have any concerns about sexuality? b. Im glad to hear you are sleeping well now. c. Sleep near your spouse in case of emergency. d. Why would you move into the guest room?

a. Do you have any concerns about sexuality? Concerns about resuming sexual activity are common after cardiac events. The nurse should gently inquire if this is the issue. While it is good that the client is sleeping well, the nurse should investigate the reason for the move. The other two responses are likely to cause the client to be defensive.

A nurse obtains the health history of a client with a suspected diagnosis of bladder cancer. Which question should the nurse ask when determining this clients risk factors? a. Do you smoke cigarettes? b. Do you use any alcohol? c. Do you use recreational drugs? d. Do you take any prescription drugs?

a. Do you smoke cigarettes? Smoking is known to be a factor that greatly increases the risk of bladder cancer. Alcohol use, recreational drug use, and prescription drug use (except medications that contain phenacetin) are not known to increase the risk of developing bladder cancer.

A client has been diagnosed with tuberculosis (TB). What action by the nurse takes highest priority? a. Educating the client on adherence to the treatment regimen b. Encouraging the client to eat a well-balanced diet c. Informing the client about follow-up sputum cultures d. Teaching the client ways to balance rest with activity

a. Educating the client on adherence to the treatment regimen The treatment regimen for TB ranges from 6 to 12 months, making adherence problematic for many people. The nurse should stress the absolute importance of following the treatment plan for the entire duration of prescribed therapy. The other options are appropriate topics to educate this client on but do not take priority.

A nurse assesses a client who is prescribed fluticasone (Flovent) and notes oral lesions. Which action should the nurse take? a. Encourage oral rinsing after fluticasone administration. b. Obtain an oral specimen for culture and sensitivity. c. Start the client on a broad-spectrum antibiotic. d. Document the finding as a known side effect.

a. Encourage oral rinsing after fluticasone administration. The drug reduces local immunity and increases the risk for local infection, especially Candida albicans. Rinsing the mouth after using the inhaler will decrease the risk for developing this infection. Use of mouthwash and broad-spectrum antibiotics is not warranted in this situation. The nurse should document the finding, but the best action to take is to have the client start rinsing his or her mouth after using fluticasone. An oral specimen for culture and sensitivity will not provide information necessary to care for this client.

The nurse is caring for four hypertensive clients. Which druglaboratory value combination should the nurse report immediately to the health care provider? a. Furosemide (Lasix)/potassium: 2.1 mEq/L b. Hydrochlorothiazide (Hydrodiuril)/potassium: 4.2 mEq/L c. Spironolactone (Aldactone)/potassium: 5.1 mEq/L d. Torsemide (Demadex)/sodium: 142 mEq/L

a. Furosemide (Lasix)/potassium: 2.1 mEq/L Lasix is a loop diuretic and can cause hypokalemia. A potassium level of 2.1 mEq/L is quite low and should be reported immediately. Spironolactone is a potassium-sparing diuretic that can cause hyperkalemia. A potassium level of 5.1 mEq/L is on the high side, but it is not as critical as the low potassium with furosemide. The other two laboratory values are normal.

A client has peripheral arterial disease (PAD). What statement by the client indicates misunderstanding about self-management activities? a. I can use a heating pad on my legs if its set on low. b. I should not cross my legs when sitting or lying down. c. I will go out and buy some warm, heavy socks to wear. d. Its going to be really hard but I will stop smoking.

a. I can use a heating pad on my legs if its set on low. Clients with PAD should never use heating pads as skin sensitivity is diminished and burns can result. The other statements show good understanding of self-management.

After teaching a client with a history of renal calculi, the nurse assesses the clients understanding. Which statement made by the client indicates a correct understanding of the teaching? a. I should drink at least 3 liters of fluid every day. b. I will eliminate all dairy or sources of calcium from my diet. c. Aspirin and aspirin-containing products can lead to stones. d. The doctor can give me antibiotics at the first sign of a stone.

a. I should drink at least 3 liters of fluid every day. Dehydration contributes to the precipitation of minerals to form a stone. Although increased intake of calcium causes hypercalcemia and leads to excessive calcium filtered into the urine, if the client is well hydrated the calcium will be excreted without issues. Dehydration increases the risk for supersaturation of calcium in the urine, which contributes to stone formation. The nurse should encourage the client to drink more fluids, not decrease calcium intake. Ingestion of aspirin or aspirin-containing products does not cause a stone. Antibiotics neither prevent nor treat a stone.

After teaching a client who has stress incontinence, the nurse assesses the clients understanding. Which statement made by the client indicates a need for additional teaching? a. I will limit my total intake of fluids. b. I must avoid drinking alcoholic beverages. c. I must avoid drinking caffeinated beverages. d. I shall try to lose about 10% of my body weight.

a. I will limit my total intake of fluids. Limiting fluids concentrates urine and can irritate tissues, leading to increased incontinence. Many people try to manage incontinence by limiting fluids. Alcoholic and caffeinated beverages are bladder stimulants. Obesity increases intra-abdominal pressure, causing incontinence.

A nurse cares for a client with a 40-year smoking history who is experiencing distended neck veins and dependent edema. Which physiologic process should the nurse correlate with this clients history and clinical manifestations? a. Increased pulmonary pressure creating a higher workload on the right side of the heart b. Exposure to irritants resulting in increased inflammation of the bronchi and bronchioles c. Increased number and size of mucus glands producing large amounts of thick mucus d. Left ventricular hypertrophy creating a decrease in cardiac output

a. Increased pulmonary pressure creating a higher workload on the right side of the heart Smoking increases pulmonary hypertension, resulting in cor pulmonale, or right-sided heart failure. Increased pressures in the lungs make it more difficult for blood to flow through the lungs. Blood backs up into the right side of the heart and then into the peripheral venous system, creating distended neck veins and dependent edema. Inflammation in bronchi and bronchioles creates an airway obstruction which manifests as wheezes. Thick mucus in the lungs has no impact on distended neck veins and edema. Left ventricular hypertrophy is associated with left heart failure and is not caused by a 40-year smoking history.

A nurse auscultates a clients lung fields. Which pathophysiologic process should the nurse associate with this breath sound? (Click the media button to hear the audio clip.) a. Inflammation of the pleura b. Constriction of the bronchioles c. Upper airway obstruction d. Pulmonary vascular edema

a. Inflammation of the pleura A pleural friction rub can be heard when the pleura is inflamed and rubbing against the lung wall. The other pathophysiologic processes would not cause a pleural friction rub. Constriction of the bronchioles may be heard as a wheeze, upper airway obstruction may be heard as stridor, and pulmonary vascular edema may be heard as crackles.

A client is taking warfarin (Coumadin) and asks the nurse if taking St. Johns wort is acceptable. What response by the nurse is best? a. No, it may interfere with the warfarin. b. There isnt any information about that. c. Why would you want to take that? d. Yes, it is a good supplement for you.

a. No, it may interfere with the warfarin. Many foods and drugs interfere with warfarin, St. Johns wort being one of them. The nurse should advise the client against taking it. The other answers are not accurate.

A nurse is teaching a larger female client about alcohol intake and how it affects hypertension. The client asks if drinking two beers a night is an acceptable intake. What answer by the nurse is best? a. No, women should only have one beer a day as a general rule. b. No, you should not drink any alcohol with hypertension. c. Yes, since you are larger, you can have more alcohol. d. Yes, two beers per day is an acceptable amount of alcohol.

a. No, women should only have one beer a day as a general rule. Alcohol intake should be limited to two drinks a day for men and one drink a day for women. A drink is classified as one beer, 1.5 ounces of hard liquor, or 5 ounces of wine. Limited alcohol intake is acceptable with hypertension. The womans size does not matter.

A nurse cares for a client admitted from a nursing home after several recent falls. What prescription should the nurse complete first? a. Obtain urine sample for culture and sensitivity. b. Administer intravenous antibiotics. c. Encourage protein intake and additional fluids. d. Consult physical therapy for gait training.

a. Obtain urine sample for culture and sensitivity. Although all interventions are or might be important, obtaining a urine sample for urinalysis takes priority. Often urinary tract infection (UTI) symptoms in older adults are atypical, and a UTI may present with new onset of confusion or falling. The urine sample should be obtained before starting antibiotics. Dietary requirements and gait training should be implemented after obtaining the urine sample.

A nurse cares for a client with infective endocarditis. Which infection control precautions should the nurse use? a. Standard Precautions b. Bleeding precautions c. Reverse isolation d. Contact isolation

a. Standard Precautions The client with infective endocarditis does not pose any specific threat of transmitting the causative organism. Standard Precautions should be used. Bleeding precautions or reverse or contact isolation is not necessary.

A nurse evaluates a client with acute glomerulonephritis (GN). Which manifestation should the nurse recognize as a positive response to the prescribed treatment? a. The client has lost 11 pounds in the past 10 days. b. The clients urine specific gravity is 1.048. c. No blood is observed in the clients urine. d. The clients blood pressure is 152/88 mm Hg.

a. The client has lost 11 pounds in the past 10 days. Fluid retention is a major feature of acute GN. This weight loss represents fluid loss, indicating that the glomeruli are performing the function of filtration. A urine specific gravity of 1.048 is high. Blood is not usually seen in GN, so this finding would be expected. A blood pressure of 152/88 mm Hg is too high; this may indicate kidney damage or fluid overload.

A nurse teaches a young female client who is prescribed amoxicillin (Amoxil) for a urinary tract infection. Which statement should the nurse include in this clients teaching? a. Use a second form of birth control while on this medication. b. You will experience increased menstrual bleeding while on this drug. c. You may experience an irregular heartbeat while on this drug. d. Watch for blood in your urine while taking this medication.

a. Use a second form of birth control while on this medication. The client should use a second form of birth control because penicillin seems to reduce the effectiveness of estrogen-containing contraceptives. She should not experience increased menstrual bleeding, an irregular heartbeat, or blood in her urine while taking the medication.

A nurse cares for a client with right-sided heart failure. The client asks, Why do I need to weigh myself every day? How should the nurse respond?a. Weight is the best indication that you are gaining or losing fluid. b. Daily weights will help us make sure that youre eating properly. c. The hospital requires that all inpatients be weighed daily. d. You need to lose weight to decrease the incidence of heart failure.

a. Weight is the best indication that you are gaining or losing fluid. Daily weights are needed to document fluid retention or fluid loss. One liter of fluid equals 2.2 pounds. The other responses do not address the importance of monitoring fluid retention or loss.

A nurse cares for an older adult client with heart failure. The client states, I dont know what to do. I dont want to be a burden to my daughter, but I cant do it alone. Maybe I should die. How should the nurse respond? a. Would you like to talk more about this? b. You are lucky to have such a devoted daughter. c. It is normal to feel as though you are a burden. d. Would you like to meet with the chaplain?

a. Would you like to talk more about this? Depression can occur in clients with heart failure, especially older adults. Having the client talk about his or her feelings will help the nurse focus on the actual problem. Open-ended statements allow the client to respond safely and honestly. The other options minimize the clients concerns and do not allow the nurse to obtain more information to provide client-centered care.

A nursing student caring for a client removes the clients oxygen as prescribed. The client is now breathing what percentage of oxygen in the room air? a. 14% b. 21% c. 28% d. 31%

b. 21% Room air is 21% oxygen.

A pulmonary nurse cares for clients who have chronic obstructive pulmonary disease (COPD). Which client should the nurse assess first? a. A 46-year-old with a 30pack-year history of smoking b. A 52-year-old in a tripod position using accessory muscles to breathe c. A 68-year-old who has dependent edema and clubbed fingers d. A 74-year-old with a chronic cough and thick, tenacious secretions

b. A 52-year-old in a tripod position using accessory muscles to breathe The client who is in a tripod position and using accessory muscles is working to breathe. This client must be assessed first to establish how well the client is breathing and provide interventions to minimize respiratory failure. The other clients are not in acute distress.

A nurse plans care for clients with urinary incontinence. Which client is correctly paired with the appropriate intervention? a. A 29-year-old client after a difficult vaginal delivery Habit training b. A 58-year-old postmenopausal client who is not taking estrogen therapy Electrical stimulation c. A 64-year-old female with Alzheimers-type senile dementia Bladder training d. A 77-year-old female who has difficulty ambulating Exercise therapy

b. A 58-year-old postmenopausal client who is not taking estrogen therapy Electrical stimulation Exercise therapy and electrical stimulation are used for clients with stress incontinence related to childbirth or low levels of estrogen after menopause. Exercise therapy increases pelvic wall strength; it does not improve ambulation. Physical therapy and a bedside commode would be appropriate interventions for the client who has difficulty ambulating. Habit training is the type of bladder training that will be most effective with cognitively impaired clients. Bladder training can be used only with a client who is alert, aware, and able to resist the urge to urinate.

A client has been taking isoniazid (INH) for tuberculosis for 3 weeks. What laboratory results need to be reported to the health care provider immediately? a. Albumin: 5.1 g/dL b. Alanine aminotransferase (ALT): 180 U/L c. Red blood cell (RBC) count: 5.2/mm3 d. White blood cell (WBC) count: 12,500/mm3

b. Alanine aminotransferase (ALT): 180 U/L INH can cause liver damage, especially if the client drinks alcohol. The ALT (one of the liver enzymes) is extremely high and needs to be reported immediately. The albumin and RBCs are normal. The WBCs are slightly high, but that would be an expected finding in a client with an infection.

A client is in the hospital after suffering a myocardial infarction and has bathroom privileges. The nurse assists the client to the bathroom and notes the clients O2 saturation to be 95%, pulse 88 beats/min, and respiratory rate 16 breaths/min after returning to bed. What action by the nurse is best?a. Administer oxygen at 2 L/min. b. Allow continued bathroom privileges. c. Obtain a bedside commode. d. Suggest the client use a bedpan.

b. Allow continued bathroom privileges. This clients physiologic parameters did not exceed normal during and after activity, so it is safe for the client to continue using the bathroom. There is no indication that the client needs oxygen, a commode, or a bedpan.

A client has a deep vein thrombosis (DVT). What comfort measure does the nurse delegate to the unlicensed assistive personnel (UAP)? a. Ambulate the client. b. Apply a warm moist pack. c. Massage the clients leg. d. Provide an ice pack.

b. Apply a warm moist pack. Warm moist packs will help with the pain of a DVT. Ambulation is not a comfort measure. Massaging the clients legs is contraindicated to prevent complications such as pulmonary embolism. Ice packs are not recommended for DVT.

A nurse assesses a client who is recovering from extracorporeal shock wave lithotripsy for renal calculi. The nurse notes an ecchymotic area on the clients right lower back. Which action should the nurse take? a. Administer fresh-frozen plasma. b. Apply an ice pack to the site. c. Place the client in the prone position. d. Obtain serum coagulation test results.

b. Apply an ice pack to the site. The shock waves from lithotripsy can cause bleeding into the tissues through which the waves pass. Application of ice can reduce the extent and discomfort of the bruising. Although coagulation test results and fresh-frozen plasma are used to assess and treat bleeding disorders, ecchymosis after this procedure is not unusual and does not warrant a higher level of intervention. Changing the clients position will not decrease bleeding.

A nurse assesses a client who has a history of heart failure. Which question should the nurse ask to assess the extent of the clients heart failure? a. Do you have trouble breathing or chest pain? b. Are you able to walk upstairs without fatigue? c. Do you awake with breathlessness during the night? d. Do you have new-onset heaviness in your legs?

b. Are you able to walk upstairs without fatigue? Clients with a history of heart failure generally have negative findings, such as shortness of breath. The nurse needs to determine whether the clients activity is the same or worse, or whether the client identifies a decrease in activity level. Trouble breathing, chest pain, breathlessness at night, and peripheral edema are symptoms of heart failure, but do not provide data that can determine the extent of the clients heart failure.

A nurse cares for a client with arthritis who reports frequent asthma attacks. Which action should the nurse take first? a. Review the clients pulmonary function test results. b. Ask about medications the client is currently taking. c. Assess how frequently the client uses a bronchodilator. d. Consult the provider and request arterial blood gases.

b. Ask about medications the client is currently taking. Aspirin and other nonsteroidal anti-inflammatory drugs (NSAIDs) can trigger asthma in some people. This results from increased production of leukotriene when aspirin or NSAIDs suppress other inflammatory pathways and is a high priority given the clients history. Reviewing pulmonary function test results will not address the immediate problem of frequent asthma attacks. This is a good intervention for reviewing response to bronchodilators. Questioning the client about the use of bronchodilators will address interventions for the attacks but not their cause. Reviewing arterial blood gas results would not be of use in a client between attacks because many clients are asymptomatic when not having attacks.

A nurse provides health screening for a community health center with a large population of African- American clients. Which priority assessment should the nurse include when working with this population? a. Measure height and weight. b. Assess blood pressure. c. Observe for any signs of abuse. d. Ask about medications.

b. Assess blood pressure. All interventions are important for the visiting nurse to accomplish. However, African Americans have a high rate of hypertension leading to end-stage renal disease. Each encounter that the nurse has with an African- American client provides a chance to detect hypertension and treat it. If the client is already on antihypertensive medication, assessing blood pressure monitors therapy.

A client is 4 hours postoperative after a femoropopliteal bypass. The client reports throbbing leg pain on the affected side, rated as 7/10. What action by the nurse takes priority? a. Administer pain medication as ordered. b. Assess distal pulses and skin color. c. Document the findings in the clients chart. d. Notify the surgeon immediately.

b. Assess distal pulses and skin color. Once perfusion has been restored or improved to an extremity, clients can often feel a throbbing pain due to the increased blood flow. However, it is important to differentiate this pain from ischemia. The nurse should assess for other signs of perfusion, such as distal pulses and skin color/temperature. Administering pain medication is done once the nurse determines the clients perfusion status is normal. Documentation needs to be thorough. Notifying the surgeon is not necessary.

A client has intra-arterial blood pressure monitoring after a myocardial infarction. The nurse notes the clients heart rate has increased from 88 to 110 beats/min, and the blood pressure dropped from 120/82 to 100/60 mm Hg. What action by the nurse is most appropriate? a. Allow the client to rest quietly. b. Assess the client for bleeding. c. Document the findings in the chart. d. Medicate the client for pain.

b. Assess the client for bleeding. A major complication related to intra-arterial blood pressure monitoring is hemorrhage from the insertion site. Since these vital signs are out of the normal range, are a change, and are consistent with blood loss, the nurse should assess the client for any bleeding associated with the arterial line. The nurse should document the findings after a full assessment. The client may or may not need pain medication and rest; the nurse first needs to rule out any emergent bleeding.

A client is in the family medicine clinic reporting a dry, sore throat. The provider asks the nurse to assess for odynophagia. What assessment technique is most appropriate? a. Ask the client what foods cause trouble swallowing. b. Assess the client for pain when swallowing. c. Determine if the client can swallow saliva. d. Palpate the clients jaw while swallowing.

b. Assess the client for pain when swallowing. Odynophagia is painful swallowing. The nurse should assess the client for this either by asking or by having the client attempt to drink water. It is not related to specific foods and is not assessed by palpating the jaw. Being unable to swallow saliva is not odynophagia, but it would be a serious situation.

A client has hypertension and high risk factors for cardiovascular disease. The client is overwhelmed with the recommended lifestyle changes. What action by the nurse is best? a. Assess the clients support system. b. Assist in finding one change the client can control. c. Determine what stressors the client faces in daily life. d. Inquire about delegating some of the clients obligations.

b. Assist in finding one change the client can control. All options are appropriate when assessing stress and responses to stress. However, this client feels overwhelmed by the suggested lifestyle changes. Instead of looking at all the needed changes, the nurse should assist the client in choosing one the client feels optimistic about controlling. Once the client has mastered that change, he or she can move forward with another change. Determining support systems, daily stressors, and delegation opportunities does not directly impact the clients feelings of control.

A nurse assesses a client who has mitral valve regurgitation. For which cardiac dysrhythmia should the nurse assess? a. Preventricular contractions b. Atrial fibrillation c. Symptomatic bradycardia d. Sinus tachycardia

b. Atrial fibrillation Atrial fibrillation is a clinical manifestation of mitral valve regurgitation and stenosis. Preventricular contractions and bradycardia are not associated with valvular problems. These are usually identified in clients with electrolyte imbalances, myocardial infarction, and sinus node problems. Sinus tachycardia is a manifestation of aortic regurgitation due to a decrease in cardiac output.

A nurse teaches a client recovering from a heart transplant who is prescribed cyclosporine (Sandimmune). Which statement should the nurse include in this clients discharge teaching? a. Use a soft-bristled toothbrush and avoid flossing. b. Avoid large crowds and people who are sick. c. Change positions slowly to avoid hypotension. d. Check your heart rate before taking the medication.

b. Avoid large crowds and people who are sick. These agents cause immune suppression, leaving the client more vulnerable to infection. The medication does not place the client at risk for bleeding, orthostatic hypotension, or a change in heart rate.

The nurse is evaluating a 3-day diet history with a client who has an elevated lipid panel. What meal selection indicates the client is managing this condition well with diet? a. A 4-ounce steak, French fries, iceberg lettuce b. Baked chicken breast, broccoli, tomatoes c. Fried catfish, cornbread, peas d. Spaghetti with meat sauce, garlic bread

b. Baked chicken breast, broccoli, tomatoes The diet recommended for this client would be low in saturated fats and red meat, high in vegetables and whole grains (fiber), low in salt, and low in trans fat. The best choice is the chicken with broccoli and tomatoes. The French fries have too much fat and the iceberg lettuce has little fiber. The catfish is fried. The spaghetti dinner has too much red meat and no vegetables.

A nurse cares for a client recovering from prosthetic valve replacement surgery. The client asks, Why will I need to take anticoagulants for the rest of my life? How should the nurse respond? a. The prosthetic valve places you at greater risk for a heart attack. b. Blood clots form more easily in artificial replacement valves. c. The vein taken from your leg reduces circulation in the leg. d. The surgery left a lot of small clots in your heart and lungs.

b. Blood clots form more easily in artificial replacement valves. Synthetic valve prostheses and scar tissue provide surfaces on which platelets can aggregate easily and initiate the formation of blood clots. The other responses are inaccurate.

After delegating care to an unlicensed assistive personnel (UAP) for a client who is prescribed habit training to manage incontinence, a nurse evaluates the UAPs understanding. Which action indicates the UAP needs additional teaching? a. Toileting the client after breakfast b. Changing the clients incontinence brief when wet c. Encouraging the client to drink fluids d. Recording the clients incontinence episodes

b. Changing the clients incontinence brief when wet Habit training is undermined by the use of absorbent incontinence briefs or pads. The nurse should re-educate the UAP on the technique of habit training. The UAP should continue to toilet the client after meals, encourage the client to drink fluids, and record incontinent episodes.

A nurse administers medications to a client who has asthma. Which medication classification is paired correctly with its physiologic response to the medication? a. Bronchodilator Stabilizes the membranes of mast cells and prevents the release of inflammatory mediators b. Cholinergic antagonist Causes bronchodilation by inhibiting the parasympathetic nervous system c. Corticosteroid Relaxes bronchiolar smooth muscles by binding to and activating pulmonary beta2 receptors d. Cromone Disrupts the production of pathways of inflammatory mediators

b. Cholinergic antagonist Causes bronchodilation by inhibiting the parasympathetic nervous system Cholinergic antagonist drugs cause bronchodilation by inhibiting the parasympathetic nervous system. This allows the sympathetic nervous system to dominate and release norepinephrine that actives beta2 receptors. Bronchodilators relax bronchiolar smooth muscles by binding to and activating pulmonary beta2 receptors. Corticosteroids disrupt the production of pathways of inflammatory mediators. Cromones stabilize the membranes of mast cells and prevent the release of inflammatory mediators.

A nurse is caring for four clients. Which one should the nurse see first? a. Client who needs a beta blocker, and has a blood pressure of 92/58 mm Hg b. Client who had a first dose of captopril (Capoten) and needs to use the bathroom c. Hypertensive client with a blood pressure of 188/92 mm Hg d. Client who needs pain medication prior to a dressing change of a surgical wound

b. Client who had a first dose of captopril (Capoten) and needs to use the bathroom Angiotensin-converting enzyme inhibitors such as captopril can cause hypotension, especially after the first dose. The nurse should see this client first to prevent falling if the client decides to get up without assistance. The two blood pressure readings are abnormal but not critical. The nurse should check on the client with higher blood pressure next to assess for problems related to the reading. The nurse can administer the beta blocker as standards state to hold it if the systolic blood pressure is below 90 mm Hg. The client who needs pain medication prior to the dressing change is not a priority over client safety and assisting the other client to the bathroom.

A nurse is in charge of the coronary intensive care unit. Which client should the nurse see first? a. Client on a nitroglycerin infusion at 5 mcg/min, not titrated in the last 4 hours b. Client who is 1 day post coronary artery bypass graft, blood pressure 180/100 mm Hg c. Client who is 1 day post percutaneous coronary intervention, going home this morning d. Client who is 2 days post coronary artery bypass graft, became dizzy this a.m. while walking

b. Client who is 1 day post coronary artery bypass graft, blood pressure 180/100 mm Hg Hypertension after coronary artery bypass graft surgery can be dangerous because it puts too much pressure on the suture lines and can cause bleeding. The charge nurse should see this client first. The client who became dizzy earlier should be seen next. The client on the nitroglycerin drip is stable. The client going home can wait until the other clients are cared for.

A nurse is caring for four clients. Which client should the nurse assess first? a. Client with an acute myocardial infarction, pulse 102 beats/min b. Client who is 1 hour post angioplasty, has tongue swelling and anxiety c. Client who is post coronary artery bypass, chest tube drained 100 mL/hr d. Client who is post coronary artery bypass, potassium 4.2 mEq/L

b. Client who is 1 hour post angioplasty, has tongue swelling and anxiety The post-angioplasty client with tongue swelling and anxiety is exhibiting manifestations of an allergic reaction that could progress to anaphylaxis. The nurse should assess this client first. The client with a heart rate of 102 beats/min may have increased oxygen demands but is just over the normal limit for heart rate. The two post coronary artery bypass clients are stable.

The charge nurse on a medical unit is preparing to admit several clients who have possible pandemic flu during a preparedness drill. What action by the nurse is best? a. Admit the clients on Contact Precautions. b. Cohort the clients in the same area of the unit. c. Do not allow pregnant caregivers to care for these clients. d. Place the clients on enhanced Droplet Precautions.

b. Cohort the clients in the same area of the unit. Preventing the spread of pandemic flu is equally important as caring for the clients who have it. Clients can be cohorted together in the same set of rooms on one part of the unit to use distancing to help prevent the spread of the disease. The other actions are not appropriate.

While assessing a client who is 12 hours postoperative after a thoracotomy for lung cancer, a nurse notices that the lower chest tube is dislodged. Which action should the nurse take first? a. Assess for drainage from the site. b. Cover the insertion site with sterile gauze. c. Contact the provider and obtain a suture kit. d. Reinsert the tube using sterile technique.

b. Cover the insertion site with sterile gauze. Immediately covering the insertion site helps prevent air from entering the pleural space and causing a pneumothorax. The area will not reseal quickly enough to prevent air from entering the chest. The nurse should not leave the client to obtain a suture kit. An occlusive dressing may cause a tension pneumothorax. The site should only be assessed after the insertion site is covered. The provider should be called to reinsert the chest tube or prescribe other treatment options.

A client is wearing a Venturi mask to deliver oxygen and the dinner tray has arrived. What action by the nurse is best? a. Assess the clients oxygen saturation and, if normal, turn off the oxygen. b. Determine if the client can switch to a nasal cannula during the meal. c. Have the client lift the mask off the face when taking bites of food. d. Turn the oxygen off while the client eats the meal and then restart it.

b. Determine if the client can switch to a nasal cannula during the meal. Oxygen is a drug that needs to be delivered constantly. The nurse should determine if the provider has approved switching to a nasal cannula during meals. If not, the nurse should consult with the provider about this issue. The oxygen should not be turned off. Lifting the mask to eat will alter the FiO2 delivered.

A client had a percutaneous transluminal coronary angioplasty for peripheral arterial disease. What assessment finding by the nurse indicates a priority outcome for this client has been met? a. Pain rated as 2/10 after medication b. Distal pulse on affected extremity 2+/4+ c. Remains on bedrest as directed d. Verbalizes understanding of procedure

b. Distal pulse on affected extremity 2+/4+ Assessing circulation distal to the puncture site is a critical nursing action. A pulse of 2+/4+ indicates good perfusion. Pain control, remaining on bedrest as directed after the procedure, and understanding are all important, but do not take priority over perfusion.

A nurse assesses a client with mitral valve stenosis. What clinical manifestation should alert the nurse to the possibility that the clients stenosis has progressed? a. Oxygen saturation of 92% b. Dyspnea on exertion c. Muted systolic murmur d. Upper extremity weakness

b. Dyspnea on exertion Dyspnea on exertion develops as the mitral valvular orifice narrows and pressure in the lungs increases. The other manifestations do not relate to the progression of mitral valve stenosis.

A client is scheduled to have a tracheostomy placed in an hour. What action by the nurse is the priority? a. Administer prescribed anxiolytic medication. b. Ensure informed consent is on the chart. c. Reinforce any teaching done previously. d. Start the preoperative antibiotic infusion.

b. Ensure informed consent is on the chart. Since this is an operative procedure, the client must sign an informed consent, which must be on the chart. Giving anxiolytics and antibiotics and reinforcing teaching may also be required but do not take priority.

A client has hemodynamic monitoring after a myocardial infarction. What safety precaution does the nurse implement for this client? a. Document pulmonary artery wedge pressure (PAWP) readings and assess their trends. b. Ensure the balloon does not remain wedged. c. Keep the client on strict NPO status. d. Maintain the client in a semi-Fowlers position.

b. Ensure the balloon does not remain wedged. If the balloon remains inflated, it can cause pulmonary infarction or rupture. The nurse should ensure the balloon remains deflated between PAWP readings. Documenting PAWP readings and assessing trends is an important nursing action related to hemodynamic monitoring, but is not specifically related to safety. The client does not have to be NPO while undergoing hemodynamic monitoring. Positioning may or may not affect readings.

A client presents to the emergency department with a severely lacerated artery. What is the priority action for the nurse? a. Administer oxygen via non-rebreather mask. b. Ensure the client has a patent airway. c. Prepare to assist with suturing the artery. d. Start two large-bore IVs with normal saline.

b. Ensure the client has a patent airway. Airway always takes priority, followed by breathing and circulation. The nurse ensures the client has a patent airway prior to providing any other care measures.

A home health care nurse is visiting an older client who lives alone after being discharged from the hospital after a coronary artery bypass graft. What finding in the home most causes the nurse to consider additional referrals? a. Dirty carpets in need of vacuuming b. Expired food in the refrigerator c. Old medications in the kitchen d. Several cats present in the home

b. Expired food in the refrigerator Expired food in the refrigerator demonstrates a safety concern for the client and a possible lack of money to buy food. The nurse can consider a referral to Meals on Wheels or another home-based food program. Dirty carpets may indicate the client has no household help and is waiting for clearance to vacuum. Old medications can be managed by the home health care nurse and the client working collaboratively. Having pets is not a cause for concern.

A client is in the family practice clinic reporting a severe cough that has lasted for 5 weeks. The client is so exhausted after coughing that work has become impossible. What action by the nurse is most appropriate? a. Arrange for immediate hospitalization. b. Facilitate polymerase chain reaction testing. c. Have the client produce a sputum sample. d. Obtain two sets of blood cultures.

b. Facilitate polymerase chain reaction testing. Polymerase chain reaction testing is used to diagnose pertussis, which this client is showing manifestations of. Hospitalization may or may not be needed but is not the most important action. The client may or may not be able to produce sputum, but sputum cultures for this disease must be obtained via deep suctioning. Blood cultures will be negative.

A nurse assesses a client with pericarditis. Which assessment finding should the nurse expect to find? a. Heart rate that speeds up and slows down b. Friction rub at the left lower sternal border c. Presence of a regular gallop rhythm d. Coarse crackles in bilateral lung bases

b. Friction rub at the left lower sternal border The client with pericarditis may present with a pericardial friction rub at the left lower sternal border. This sound is the result of friction from inflamed pericardial layers when they rub together. The other assessments are not related.

A nurse teaches a client with heart failure about energy conservation. Which statement should the nurse include in this clients teaching? a. Walk until you become short of breath, and then walk back home. b. Gather everything you need for a chore before you begin. c. Pull rather than push or carry items heavier than 5 pounds. d. Take a walk after dinner every day to build up your strength.

b. Gather everything you need for a chore before you begin. A client who has heart failure should be taught to conserve energy. Gathering all supplies needed for a chore at one time decreases the amount of energy needed. The client should not walk until becoming short of breath because he or she may not make it back home. Pushing a cart takes less energy than pulling or lifting. Although walking after dinner may help the client, the nurse should teach the client to complete activities when he or she has the most energy. This is usually in the morning.

A client has presented to the emergency department with an acute myocardial infarction (MI). What action by the nurse is best to meet The Joint Commissions Core Measures outcomes? a. Obtain an electrocardiogram (ECG) now and in the morning. b. Give the client an aspirin. c. Notify the Rapid Response Team. d. Prepare to administer thrombolytics.

b. Give the client an aspirin. The Joint Commissions Core Measures set for acute MI require that aspirin is administered when a client with MI presents to the emergency department or when an MI occurs in the hospital. A rapid ECG is vital, but getting another one in the morning is not part of the Core Measures set. The Rapid Response Team is not needed if an emergency department provider is available. Thrombolytics may or may not be needed.

After teaching a client with early polycystic kidney disease (PKD) about nutritional therapy, the nurse assesses the clients understanding. Which statement made by the client indicates a correct understanding of the teaching? a. I will take a laxative every night before going to bed. b. I must increase my intake of dietary fiber and fluids. c. I shall only use salt when I am cooking my own food. d. Ill eat white bread to minimize gastrointestinal gas.

b. I must increase my intake of dietary fiber and fluids. Clients with PKD often have constipation, which can be managed with increased fiber, exercise, and drinking plenty of water. Laxatives should be used cautiously. Clients with PKD should be on a restricted salt diet, which includes not cooking with salt. White bread has a low fiber count and would not be included in a high- fiber diet.

After teaching a client with renal cancer who is prescribed temsirolimus (Torisel), the nurse assesses the clients understanding. Which statement made by the client indicates a correct understanding of the teaching? a. I will take this medication with food and plenty of water. b. I shall keep my appointment at the infusion center each week. c. Ill limit my intake of green leafy vegetables while on this medication. d. I must not take this medication if I have an infection or am feeling ill.

b. I shall keep my appointment at the infusion center each week. Temsirolimus is administered as a weekly intravenous infusion. This medication blocks protein that is needed for cell division and therefore inhibits cell cycle progression. This medication is not taken orally, and clients do not need to follow a specific diet.

After teaching a client who is being discharged home after mitral valve replacement surgery, the nurse assesses the clients understanding. Which client statement indicates a need for additional teaching? a. Ill be able to carry heavy loads after 6 months of rest. b. I will have my teeth cleaned by my dentist in 2 weeks. c. I must avoid eating foods high in vitamin K, like spinach. d. I must use an electric razor instead of a straight razor to shave.

b. I will have my teeth cleaned by my dentist in 2 weeks. Clients who have defective or repaired valves are at high risk for endocarditis. The client who has had valve surgery should avoid dental procedures for 6 months because of the risk for endocarditis. When undergoing a mitral valve replacement surgery, the client needs to be placed on anticoagulant therapy to prevent vegetation forming on the new valve. Clients on anticoagulant therapy should be instructed on bleeding precautions, including using an electric razor. If the client is prescribed warfarin, the client should avoid foods high in vitamin K. Clients recovering from open heart valve replacements should not carry anything heavy for 6 months while the chest incision and muscle heal.

After teaching a client with nephrotic syndrome and a normal glomerular filtration, the nurse assesses the clients understanding. Which statement made by the client indicates a correct understanding of the nutritional therapy for this condition? a. I must decrease my intake of fat. b. I will increase my intake of protein. c. A decreased intake of carbohydrates will be required. d. An increased intake of vitamin C is necessary.

b. I will increase my intake of protein. In nephrotic syndrome, the renal loss of protein is significant, leading to hypoalbuminemia and edema formation. If glomerular filtration is normal or near normal, increased protein loss should be matched by increased intake of protein. The client would not need to adjust fat, carbohydrates, or vitamins based on this disorder.

After teaching a client with hypertension secondary to renal disease, the nurse assesses the clients understanding. Which statement made by the client indicates a need for additional teaching? a. I can prevent more damage to my kidneys by managing my blood pressure. b. If I have increased urination at night, I need to drink less fluid during the day. c. I need to see the registered dietitian to discuss limiting my protein intake. d. It is important that I take my antihypertensive medications as directed.

b. If I have increased urination at night, I need to drink less fluid during the day. The client should not restrict fluids during the day due to increased urination at night. Clients with renal disease may be prescribed fluid restrictions. These clients should be assessed thoroughly for potential dehydration. Increased nocturnal voiding can be decreased by consuming fluids earlier in the day. Blood pressure control is needed to slow the progression of renal dysfunction. When dietary protein is restricted, refer the client to the registered dietitian as needed.

A client seen in the emergency department reports fever, fatigue, and dry cough but no other upper respiratory symptoms. A chest x-ray reveals mediastinal widening. What action by the nurse is best? a. Collect a sputum sample for culture by deep suctioning. b. Inform the client that antibiotics will be needed for 60 days. c. Place the client on Airborne Precautions immediately. d. Tell the client that directly observed therapy is needed.

b. Inform the client that antibiotics will be needed for 60 days. This client has manifestations of early inhalation anthrax. For treatment, after IV antibiotics are finished, oral antibiotics are continued for at least 60 days. Sputum cultures are not needed. Anthrax is not transmissible from person to person, so Standard Precautions are adequate. Directly observed therapy is often used for tuberculosis.

An emergency department nurse assesses a client with kidney trauma and notes that the clients abdomen is tender and distended and blood is visible at the urinary meatus. Which prescription should the nurse consult the provider about before implementation? a. Assessing vital signs every 15 minutes b. Inserting an indwelling urinary catheter c. Administering intravenous fluids at 125 mL/hr d. Typing and crossmatching for blood products

b. Inserting an indwelling urinary catheter Clients with blood at the urinary meatus should not have a urinary catheter inserted via the urethra before additional diagnostic studies are done. The urethra could be torn. The nurse should question the provider about the need for a catheter; if one is needed, the provider can insert a suprapubic catheter. The nurse should monitor the clients vital signs closely, send blood for type and crossmatch in case the client needs blood products, and administer intravenous fluids.

After administering newly prescribed captopril (Capoten) to a client with heart failure, the nurse implements interventions to decrease complications. Which priority intervention should the nurse implement for this client? a. Provide food to decrease nausea and aid in absorption. b. Instruct the client to ask for assistance when rising from bed. c. Collaborate with unlicensed assistive personnel to bathe the client. d. Monitor potassium levels and check for symptoms of hypokalemia.

b. Instruct the client to ask for assistance when rising from bed. Administration of the first dose of angiotensin-converting enzyme (ACE) inhibitors is often associated with hypotension, usually termed first-dose effect. The nurse should instruct the client to seek assistance before arising from bed to prevent injury from postural hypotension. ACE inhibitors do not need to be taken with food. Collaboration with unlicensed assistive personnel to provide hygiene is not a priority. The client should be encouraged to complete activities of daily living as independently as possible. The nurse should monitor for hyperkalemia, not hypokalemia, especially if the client has renal insufficiency secondary to heart failure.

A nurse is caring for a client using oxygen while in the hospital. What assessment finding indicates that goals for a priority diagnosis are being met? a. 100% of meals being eaten by the client b. Intact skin behind the ears c. The client understanding the need for oxygen d. Unchanged weight for the past 3 days

b. Intact skin behind the ears Oxygen tubing can cause pressure ulcers, so clients using oxygen have the nursing diagnosis of Risk for Impaired Skin Integrity. Intact skin behind the ears indicates that goals for this diagnosis are being met. Nutrition and weight are not related to using oxygen. Understanding the need for oxygen is important but would not take priority over a physical problem.

A nurse cares for a postmenopausal client who has had two episodes of bacterial urethritis in the last 6 months. The client asks, I never have urinary tract infections. Why is this happening now? How should the nurse respond? a. Your immune system becomes less effective as you age. b. Low estrogen levels can make the tissue more susceptible to infection. c. You should be more careful with your personal hygiene in this area. d. It is likely that you have an untreated sexually transmitted disease.

b. Low estrogen levels can make the tissue more susceptible to infection. Low estrogen levels decrease moisture and secretions in the perineal area and cause other tissue changes, predisposing it to the development of infection. Urethritis is most common in postmenopausal women for this reason. Although immune function does decrease with aging and sexually transmitted diseases are a known cause of urethritis, the most likely reason in this client is low estrogen levels. Personal hygiene usually does not contribute to this disease process.

A client has a tracheostomy tube in place. When the nurse suctions the client, food particles are noted. What action by the nurse is best? a. Elevate the head of the clients bed. b. Measure and compare cuff pressures. c. Place the client on NPO status. d. Request that the client have a swallow study.

b. Measure and compare cuff pressures. Constant pressure from the tracheostomy tube cuff can cause tracheomalacia, leading to dilation of the tracheal passage. This can be manifested by food particles seen in secretions or by noting that larger and larger amounts of pressure are needed to keep the tracheostomy cuff inflated. The nurse should measure the pressures and compare them to previous ones to detect a trend. Elevating the head of the bed, placing the client on NPO status, and requesting a swallow study will not correct this situation.

A client has been diagnosed with hypertension but does not take the antihypertensive medications because of a lack of symptoms. What response by the nurse is best? a. Do you have trouble affording your medications? b. Most people with hypertension do not have symptoms. c. You are lucky; most people get severe morning headaches. d. You need to take your medicine or you will get kidney failure.

b. Most people with hypertension do not have symptoms. Most people with hypertension are asymptomatic, although a small percentage do have symptoms such as headache. The nurse should explain this to the client. Asking about paying for medications is not related because the client has already admitted nonadherence. Threatening the client with possible complications will not increase compliance.

A nurse assesses a client admitted to the cardiac unit. Which statement by the client alerts the nurse to the possibility of right-sided heart failure? a. I sleep with four pillows at night. b. My shoes fit really tight lately. c. I wake up coughing every night. d. I have trouble catching my breath.

b. My shoes fit really tight lately. Signs of systemic congestion occur with right-sided heart failure. Fluid is retained, pressure builds in the venous system, and peripheral edema develops. Left-sided heart failure symptoms include respiratory symptoms. Orthopnea, coughing, and difficulty breathing all could be results of left-sided heart failure.

A client is receiving an infusion of alteplase (Activase) for an intra-arterial clot. The client begins to mumble and is disoriented. What action by the nurse takes priority? a. Assess the clients neurologic status. b. Notify the Rapid Response Team. c. Prepare to administer vitamin K. d. Turn down the infusion rate.

b. Notify the Rapid Response Team. Clients on fibrinolytic therapy are at high risk of bleeding. The sudden onset of neurologic signs may indicate the client is having a hemorrhagic stroke. The nurse does need to complete a thorough neurological examination, but should first call the Rapid Response Team based on the clients manifestations. The nurse notifies the Rapid Response Team first. Vitamin K is not the antidote for this drug. Turning down the infusion rate will not be helpful if the client is still receiving any of the drug.

A nurse reviews the laboratory findings of a client with a urinary tract infection. The laboratory report notes a shift to the left in a clients white blood cell count. Which action should the nurse take? a. Request that the laboratory perform a differential analysis on the white blood cells. b. Notify the provider and start an intravenous line for parenteral antibiotics. c. Collaborate with the unlicensed assistive personnel (UAP) to strain the clients urine for renal calculi. d. Assess the client for a potential allergic reaction and anaphylactic shock.

b. Notify the provider and start an intravenous line for parenteral antibiotics. An increase in band cells creates a shift to the left. A left shift most commonly occurs with urosepsis and is seen rarely with uncomplicated urinary tract infections. The nurse will be administering antibiotics, most likely via IV, so he or she should notify the provider and prepare to give the antibiotics. The shift to the left is part of a differential white blood cell count. The nurse would not need to strain urine for stones. Allergic reactions are associated with elevated eosinophil cells, not band cells.

The nurse is caring for a client with a chest tube after a coronary artery bypass graft. The drainage slows significantly. What action by the nurse is most important? a. Increase the setting on the suction. b. Notify the provider immediately. c. Re-position the chest tube. d. Take the tubing apart to assess for clots.

b. Notify the provider immediately. If the drainage in the chest tube decreases significantly and dramatically, the tube may be blocked by a clot. This could lead to cardiac tamponade. The nurse should notify the provider immediately. The nurse should not independently increase the suction, re-position the chest tube, or take the tubing apart.

An older adult is brought to the emergency department by a family member, who reports a moderate change in mental status and mild cough. The client is afebrile. The health care provider orders a chest x-ray. The family member questions why this is needed since the manifestations seem so vague. What response by the nurse is best? a. Chest x-rays are always ordered when we suspect pneumonia. b. Older people often have vague symptoms, so an x-ray is essential. c. The x-ray can be done and read before laboratory work is reported. d. We are testing for any possible source of infection in the client.

b. Older people often have vague symptoms, so an x-ray is essential. It is essential to obtain an early chest x-ray in older adults suspected of having pneumonia because symptoms are often vague. Waiting until definitive manifestations are present to obtain the x-ray leads to a costly delay in treatment. Stating that chest x-rays are always ordered does not give the family definitive information. The x- ray can be done while laboratory values are still pending, but this also does not provide specific information about the importance of a chest x-ray in this client. The client has manifestations of pneumonia, so the staff is not testing for any possible source of infection but rather is testing for a suspected disorder.

A nurse assesses a male client who is recovering from a urologic procedure. Which assessment finding indicates an obstruction of urine flow? a. Severe pain b. Overflow incontinence c. Hypotension d. Blood-tinged urine

b. Overflow incontinence The most common manifestation of urethral stricture after a urologic procedure is obstruction of urine flow. This rarely causes pain and has no impact on blood pressure. The client may experience overflow incontinence with the involuntary loss of urine when the bladder is distended. Blood in the urine is not a manifestation of the obstruction of urine flow.

A nurse is caring for a client with a deep vein thrombosis (DVT). What nursing assessment indicates a priority outcome has been met? a. Ambulates with assistance b. Oxygen saturation of 98% c. Pain of 2/10 after medication d. Verbalizing risk factors

b. Oxygen saturation of 98% A critical complication of DVT is pulmonary embolism. A normal oxygen saturation indicates that this has not occurred. The other assessments are also positive, but not the priority.

A nurse wants to provide community service that helps meet the goals of Healthy People 2020 (HP2020) related to cardiovascular disease and stroke. What activity would best meet this goal? a. Teach high school students heart-healthy living. b. Participate in blood pressure screenings at the mall. c. Provide pamphlets on heart disease at the grocery store. d. Set up an Ask the nurse booth at the pet store.

b. Participate in blood pressure screenings at the mall. An important goal of HP2020 is to increase the proportion of adults who have had their blood pressure measured within the preceding 2 years and can state whether their blood pressure was normal or high. Participating in blood pressure screening in a public spot will best help meet that goal. The other options are all appropriate but do not specifically help meet a goal.

1. A nurse assesses a client with polycystic kidney disease (PKD). Which assessment finding should alert the nurse to immediately contact the health care provider? a. Flank pain b. Periorbital edema c. Bloody and cloudy urine d. Enlarged abdomen

b. Periorbital edema Periorbital edema would not be a finding related to PKD and should be investigated further. Flank pain and a distended or enlarged abdomen occur in PKD because the kidneys enlarge and displace other organs. Urine can be bloody or cloudy as a result of cyst rupture or infection.

Which teaching point is most important for the client with bacterial pharyngitis? a. Gargle with warm salt water. b. Take all antibiotics as directed. c. Use a humidifier in the bedroom. d. Wash hands frequently.

b. Take all antibiotics as directed. Any client on antibiotics must be instructed to complete the entire course of antibiotics. Not completing them can lead to complications or drug-resistant strains of bacteria. The other instructions are appropriate, just not the most important.

A client with coronary artery disease (CAD) asks the nurse about taking fish oil supplements. What response by the nurse is best? a. Fish oil is contraindicated with most drugs for CAD. b. The best source is fish, but pills have benefits too. c. There is no evidence to support fish oil use with CAD. d. You can reverse CAD totally with diet and supplements.

b. The best source is fish, but pills have benefits too. Omega-3 fatty acids have shown benefit in reducing lipid levels, in reducing the incidence of sudden cardiac death, and for stabilizing atherosclerotic plaque. The best source is fish three times a week or some fish oil supplements. The other options are not accurate.

A client with a new tracheostomy is being seen in the oncology clinic. What finding by the nurse best indicates that goals for the nursing diagnosis Impaired Self-Esteem are being met? a. The client demonstrates good understanding of stoma care. b. The client has joined a book club that meets at the library. c. Family members take turns assisting with stoma care. d. Skin around the stoma is intact without signs of infection.

b. The client has joined a book club that meets at the library. The client joining a book club that meets outside the home and requires him or her to go out in public is the best sign that goals for Impaired Self-Esteem are being met. The other findings are all positive signs but do not relate to this nursing diagnosis.

After teaching a client how to perform diaphragmatic breathing, the nurse assesses the clients understanding. Which action demonstrates that the client correctly understands the teaching?a. The client lays on his or her side with his or her knees bent. b. The client places his or her hands on his or her abdomen. c. The client lays in a prone position with his or her legs straight. d. The client places his or her hands above his or her head.

b. The client places his or her hands on his or her abdomen. To perform diaphragmatic breathing correctly, the client should place his or her hands on his or her abdomen to create resistance. This type of breathing cannot be performed effectively while lying on the side or with hands over the head. This type of breathing would not be as effective lying prone.

A client received tissue plasminogen activator (t-PA) after a myocardial infarction and now is on an intravenous infusion of heparin. The clients spouse asks why the client needs this medication. What response by the nurse is best? a. The t-PA didnt dissolve the entire coronary clot. b. The heparin keeps that artery from getting blocked again. c. Heparin keeps the blood as thin as possible for a longer time. d. The heparin prevents a stroke from occurring as the t-PA wears off.

b. The heparin keeps that artery from getting blocked again. After the original intracoronary clot has dissolved, large amounts of thrombin are released into the bloodstream, increasing the chance of the vessel reoccluding. The other statements are not accurate. Heparin is not a blood thinner, although laypeople may refer to it as such.

A nurse teaches a client who is starting urinary bladder training. Which statement should the nurse include in this clients teaching?a. Use the toilet when you first feel the urge, rather than at specific intervals. b. Try to consciously hold your urine until the scheduled toileting time. c. Initially try to use the toilet at least every half hour for the first 24 hours. d. The toileting interval can be increased once you have been continent for a week.

b. Try to consciously hold your urine until the scheduled toileting time. The client should try to hold the urine consciously until the next scheduled toileting time. Toileting should occur at specific intervals during the training. The toileting interval should be no less than every hour. The interval can be increased once the client becomes comfortable with the interval.

An emergency department nurse assesses a client with a history of urinary incontinence who presents with extreme dry mouth, constipation, and an inability to void. Which question should the nurse ask first? a. Are you drinking plenty of water? b. What medications are you taking? c. Have you tried laxatives or enemas? d. Has this type of thing ever happened before?

b. What medications are you taking? Some types of incontinence are treated with anticholinergic medications such as propantheline (Pro-Banthine). Anticholinergic side effects include dry mouth, constipation, and urinary retention. The nurse needs to assess the clients medication list to determine whether the client is taking an anticholinergic medication. If he or she is taking anticholinergics, the nurse should further assess the clients manifestations to determine if they are related to a simple side effect or an overdose. The other questions are not as helpful to understanding the current situation.

A client presents to the emergency department with an acute myocardial infarction (MI) at 1500 (3:00 PM). The facility has 24-hour catheterization laboratory abilities. To meet The Joint Commissions Core Measures set, by what time should the client have a percutaneous coronary intervention performed? a. 1530 (3:30 PM) b. 1600 (4:00 PM) c. 1630 (4:30 PM) d. 1700 (5:00 PM)

c. 1630 (4:30 PM) The Joint Commissions Core Measures set for MI includes percutaneous coronary intervention within 90 minutes of diagnosis of myocardial infarction. Therefore, the client should have a percutaneous coronary intervention performed no later than 1630 (4:30 PM).

The nurse instructs a client on how to correctly use an inhaler with a spacer. In which order should these steps occur? 1. Press down firmly on the canister to release one dose of medication. 2. Breathe in slowly and deeply. 3. Shake the whole unit vigorously three or four times. 4. Insert the mouthpiece of the inhaler into the nonmouthpiece end of the spacer. 5. Place the mouthpiece into your mouth, over the tongue, and seal your lips tightly around the mouthpiece. 6. Remove the mouthpiece from your mouth, keep your lips closed, and hold your breath for at least 10 seconds. a. 2, 3, 4, 5, 6, 1 b. 3, 4, 5, 1, 6, 2 c. 4, 3, 5, 1, 2, 6 d. 5, 3, 6, 1, 2, 4

c. 4, 3, 5, 1, 2, 6 The proper order for correctly using an inhaler with a spacer is as follows. Insert the mouthpiece of the inhaler into the nonmouthpiece end of the spacer. Shake the whole unit vigorously three or four times. Place the mouthpiece into the mouth, over the tongue, and seal the lips tightly around it. Press down firmly on the canister of the inhaler to release one dose of medication into the spacer. Breathe in slowly and deeply. Remove the mouthpiece from the mouth, and, keeping the lips closed, hold the breath for at least 10 seconds. Then breathe out slowly. Wait at least 1 minute between puffs.

A nurse assesses clients on the medical-surgical unit. Which client is at greatest risk for the development of bacterial cystitis? a. A 36-year-old female who has never been pregnant b. A 42-year-old male who is prescribed cyclophosphamide c. A 58-year-old female who is not taking estrogen replacement d. A 77-year-old male with mild congestive heart failure

c. A 58-year-old female who is not taking estrogen replacement Females at any age are more susceptible to cystitis than men because of the shorter urethra in women. Postmenopausal women who are not on hormone replacement therapy are at increased risk for bacterial cystitis because of changes in the cells of the urethra and vagina. The middle-aged woman who has never been pregnant would not have a risk potential as high as the older woman who is not using hormone replacement therapy.

After teaching a client with bacterial cystitis who is prescribed phenazopyridine (Pyridium), the nurse assesses the clients understanding. Which statement made by the client indicates a correct understanding of the teaching? a. I will not take this drug with food or milk. b. If I think I am pregnant, I will stop the drug. c. An orange color in my urine should not alarm me. d. I will drink two glasses of cranberry juice daily.

c. An orange color in my urine should not alarm me. Phenazopyridine discolors urine, most commonly to a deep reddish orange. Many clients think they have blood in their urine when they see this. In addition, the urine can permanently stain clothing. Phenazopyridine is safe to take if the client is pregnant. There are no dietary restrictions or needs while taking this medication.

A confused client with pneumonia is admitted with an indwelling catheter in place. During interdisciplinary rounds the following day, which question should the nurse ask the primary health care provider? a. Do you want daily weights on this client? b. Will the client be able to return home? c. Can we discontinue the indwelling catheter? d. Should we get another chest x-ray today?

c. Can we discontinue the indwelling catheter? An indwelling catheter dramatically increases the risks of urinary tract infection and urosepsis. Nursing staff should ensure that catheters are left in place only as long as they are medically needed. The nurse should inquire about removing the catheter. All other questions might be appropriate, but because of client safety, this question takes priority.z

A nurse cares for a client with chronic obstructive pulmonary disease (COPD) who appears thin and disheveled. Which question should the nurse ask first? a. Do you have a strong support system? b. What do you understand about your disease? c. Do you experience shortness of breath with basic activities? d. What medications are you prescribed to take each day?

c. Do you experience shortness of breath with basic activities? Clients with severe COPD may not be able to perform daily activities, including bathing and eating, because of excessive shortness of breath. The nurse should ask the client if shortness of breath is interfering with basic activities. Although the nurse should know about the clients support systems, current knowledge, and medications, these questions do not address the clients appearance.

A nurse cares for a middle-aged female client with diabetes mellitus who is being treated for the third episode of acute pyelonephritis in the past year. The client asks, What can I do to help prevent these infections? How should the nurse respond? a. Test your urine daily for the presence of ketone bodies and proteins. b. Use tampons rather than sanitary napkins during your menstrual period. c. Drink more water and empty your bladder more frequently during the day. d. Keep your hemoglobin A1c under 9% by keeping your blood sugar controlled.

c. Drink more water and empty your bladder more frequently during the day. Clients with long-standing diabetes mellitus are at risk for pyelonephritis for many reasons. Chronically elevated blood glucose levels spill glucose into the urine, changing the pH and providing a favorable climate for bacterial growth. The neuropathy associated with diabetes reduces bladder tone and reduces the clients sensation of bladder fullness. Thus, even with large amounts of urine, the client voids less frequently, allowing stasis and overgrowth of microorganisms. Increasing fluid intake (specifically water) and voiding frequently prevent stasis and bacterial overgrowth. Testing urine and using tampons will not help prevent pyelonephritis. A hemoglobin A1c of 9% is too high.

A nurse is teaching a client who has cystic fibrosis (CF). Which statement should the nurse include in this clients teaching? a. Take an antibiotic each day. b. Contact your provider to obtain genetic screening. c. Eat a well-balanced, nutritious diet. d. Plan to exercise for 30 minutes every day.

c. Eat a well-balanced, nutritious diet. Clients with CF often are malnourished due to vitamin deficiency and pancreatic malfunction. Maintaining nutrition is essential. Daily antibiotics and daily exercise are not essential actions. Genetic screening would not help the client manage CF better.

A nurse working in a geriatric clinic sees clients with cold symptoms and rhinitis. Which drug would be appropriate to teach these clients to take for their symptoms? a. Chlorpheniramine (Chlor- Trimeton) b. Diphenhydramine (Benadryl) c. Fexofenadine (Allegra) d. Hydroxyzine (Vistaril)

c. Fexofenadine (Allegra) First-generation antihistamines are not appropriate for use in the older population. These drugs include chlorpheniramine, diphenhydramine, and hydroxyzine. Fexofenadine is a second-generation antihistamine.

A nurse cares for a client with urinary incontinence. The client states, I am so embarrassed. My bladder leaks like a young childs bladder. How should the nurse respond? a. I understand how you feel. I would be mortified. b. Incontinence pads will minimize leaks in public. c. I can teach you strategies to help control your incontinence. d. More women experience incontinence than you might think.

c. I can teach you strategies to help control your incontinence. The nurse should accept and acknowledge the clients concerns, and assist the client to learn techniques that will allow control of urinary incontinence. The nurse should not diminish the clients concerns with the use of pads or stating statistics about the occurrence of incontinence.

A nurse assesses a client in an outpatient clinic. Which statement alerts the nurse to the possibility of left- sided heart failure? a. I have been drinking more water than usual. b. I am awakened by the need to urinate at night. c. I must stop halfway up the stairs to catch my breath. d. I have experienced blurred vision on several occasions.

c. I must stop halfway up the stairs to catch my breath. Clients with left-sided heart failure report weakness or fatigue while performing normal activities of daily living, as well as difficulty breathing, or catching their breath. This occurs as fluid moves into the alveoli. Nocturia is often seen with right-sided heart failure. Thirst and blurred vision are not related to heart failure.

The nurse is teaching a client with chronic obstructive pulmonary disease who has been prescribed continuous oxygen therapy at home. Which statement indicates the client correctly understands the teaching? a. I plan to wear my oxygen when I exercise and feel short of breath. b. I will use my portable oxygen when grilling burgers in the backyard. c. I plan to use cotton balls to cushion the oxygen tubing on my ears. d. I will only smoke while I am wearing my oxygen via nasal cannula.

c. I plan to use cotton balls to cushion the oxygen tubing on my ears. Cotton balls can decrease pressure ulcers from the oxygen tubing. Continuous oxygen orders mean the client should wear the oxygen at all times. Oxygen fuels a fire. Wearing oxygen while grilling and smoking increases the risk for fire.

After teaching a client who is prescribed a long-acting beta2 agonist medication, a nurse assesses the clients understanding. Which statement indicates the client comprehends the teaching?a. I will carry this medication with me at all times in case I need it. b. I will take this medication when I start to experience an asthma attack. c. I will take this medication every morning to help prevent an acute attack. d. I will be weaned off this medication when I no longer need it.

c. I will take this medication every morning to help prevent an acute attack. Long-acting beta2 agonist medications will help prevent an acute asthma attack because they are long acting. The client will take this medication every day for best effect. The client does not have to always keep this medication with him or her because it is not used as a rescue medication. This is not the medication the client will use during an acute asthma attack because it does not have an immediate onset of action. The client will not be weaned off this medication because this is likely to be one of his or her daily medications.

A nurse has educated a client on isoniazid (INH). What statement by the client indicates teaching has been effective? a. I need to take extra vitamin C while on INH. b. I should take this medicine with milk or juice. c. I will take this medication on an empty stomach. d. My contact lenses will be permanently stained.

c. I will take this medication on an empty stomach. INH needs to be taken on an empty stomach, either 1 hour before or 2 hours after meals. Extra vitamin B needs to be taken while on the drug. Staining of contact lenses commonly occurs while taking rifampin (Rifadin).

After teaching a client who is prescribed salmeterol (Serevent), the nurse assesses the clients understanding. Which statement by the client indicates a need for additional teaching?a. I will be certain to shake the inhaler well before I use it.b. It may take a while before I notice a change in my asthma. c. I will use the drug when I have an asthma attack. d. I will be careful not to let the drug escape out of my nose and mouth.

c. I will use the drug when I have an asthma attack. Salmeterol is designed to prevent an asthma attack; it does not relieve or reverse symptoms. Salmeterol has a slow onset of action; therefore, it should not be used as a rescue drug. The drug must be shaken well because it has a tendency to separate easily. Poor technique on the clients part allows the drug to escape through the nose and mouth.

The nurse assesses the client using the device pictured below to deliver 50% O2: The nurse finds the mask fits snugly, the skin under the mask and straps is intact, and the flow rate of the oxygen is 3 L/min. What action by the nurse is best? a. Assess the clients oxygen saturation. b. Document these findings in the chart. c. Immediately increase the flow rate. d. Turn the flow rate down to 2 L/min.

c. Immediately increase the flow rate. For the Venturi mask to deliver high flow of oxygen, the flow rate must be set correctly, usually between 4 and 10 L/min. The clients flow rate is too low and the nurse should increase it. After increasing the flow rate, the nurse assesses the oxygen saturation and documents the findings.

A client is admitted with suspected pneumonia from the emergency department. The client went to the primary care provider a few days ago and shows the nurse the results of what the client calls an allergy test, as shown below: What action by the nurse takes priority? a. Assess the client for possible items to which he or she is allergic. b. Call the primary care providers office to request records. c. Immediately place the client on Airborne Precautions. d. Prepare to begin administration of intravenous antibiotics.

c. Immediately place the client on Airborne Precautions. This allergy test is actually a positive tuberculosis test. The client should be placed on Airborne Precautions immediately. The other options do not take priority over preventing the spread of the disease.

The provider requests the nurse start an infusion of an inotropic agent on a client. How does the nurse explain the action of these drugs to the client and spouse? a. It constricts vessels, improving blood flow. b. It dilates vessels, which lessens the work of the heart. c. It increases the force of the hearts contractions. d. It slows the heart rate down for better filling.

c. It increases the force of the hearts contractions. A positive inotrope is a medication that increases the strength of the hearts contractions. The other options are not correct.

A student nurse asks what essential hypertension is. What response by the registered nurse is best? a. It means it is caused by another disease. b. It means it is essential that it be treated. c. It is hypertension with no specific cause. d. It refers to severe and life-threatening hypertension.

c. It is hypertension with no specific cause. Essential hypertension is the most common type of hypertension and has no specific cause such as an underlying disease process. Hypertension that is due to another disease process is called secondary hypertension. A severe, life-threatening form of hypertension is malignant hypertension.

The nurse is caring for a client with lung cancer who states, I dont want any pain medication because I am afraid Ill become addicted. How should the nurse respond? a. I will ask the provider to change your medication to a drug that is less potent. b. Would you like me to use music therapy to distract you from your pain? c. It is unlikely you will become addicted when taking medicine for pain. d. Would you like me to give you acetaminophen (Tylenol) instead?

c. It is unlikely you will become addicted when taking medicine for pain. Clients should be encouraged to take their pain medications; addiction usually is not an issue with a client in pain. The nurse would not request that the pain medication be changed unless it was not effective. Other methods to decrease pain can be used, in addition to pain medication.

A nurse cares for a client who is infected with Burkholderia cepacia. Which action should the nurse take first when admitting this client to a pulmonary care unit? a. Instruct the client to wash his or her hands after contact with other people. b. Implement Droplet Precautions and don a surgical mask. c. Keep the client isolated from other clients with cystic fibrosis. d. Obtain blood, sputum, and urine culture specimens.

c. Keep the client isolated from other clients with cystic fibrosis. Burkholderia cepacia infection is spread through casual contact between cystic fibrosis clients, thus the need for these clients to be separated from one another. Strict isolation measures will not be necessary. Although the client should wash his or her hands frequently, the most important measure that can be implemented on the unit is isolation of the client from other clients with cystic fibrosis. There is no need to implement Droplet Precautions or don a surgical mask when caring for this client. Obtaining blood, sputum, and urine culture specimens will not provide information necessary to care for a client with Burkholderia cepacia infection.

A client in the cardiac stepdown unit reports severe, crushing chest pain accompanied by nausea and vomiting. What action by the nurse takes priority? a. Administer an aspirin. b. Call for an electrocardiogram (ECG). c. Maintain airway patency. d. Notify the provider.

c. Maintain airway patency. Airway always is the priority. The other actions are important in this situation as well, but the nurse should stay with the client and ensure the airway remains patent (especially if vomiting occurs) while another person calls the provider (or Rapid Response Team) and facilitates getting an ECG done. Aspirin will probably be administered, depending on the providers prescription and the clients current medications.

A nurse provides phone triage to a pregnant client. The client states, I am experiencing a burning pain when I urinate. How should the nurse respond? a. This means labor will start soon. Prepare to go to the hospital. b. You probably have a urinary tract infection. Drink more cranberry juice. c. Make an appointment with your provider to have your infection treated. d. Your pelvic wall is weakening. Pelvic muscle exercises should help.

c. Make an appointment with your provider to have your infection treated. Pregnant clients with a urinary tract infection require prompt and aggressive treatment because cystitis can lead to acute pyelonephritis during pregnancy. The nurse should encourage the client to make an appointment and have the infection treated. Burning pain when urinating does not indicate the start of labor or weakening of pelvic muscles.

A nurse is assessing an obese client in the clinic for follow-up after an episode of deep vein thrombosis. The client has lost 20 pounds since the last visit. What action by the nurse is best? a. Ask if the weight loss was intended. b. Encourage a high-protein, high-fiber diet. c. Measure for new compression stockings. d. Review a 3-day food recall diary.

c. Measure for new compression stockings. Compression stockings must fit correctly in order to work. After losing a significant amount of weight, the client should be re-measured and new stockings ordered if needed. The other options are appropriate, but not the most important.

A nurse cares for a client who has developed esophagitis after undergoing radiation therapy for lung cancer. Which diet selection should the nurse provide for this client? a. Spaghetti with meat sauce, ice cream b. Chicken soup, grilled cheese sandwich c. Omelet, soft whole wheat bread d. Pasta salad, custard, orange juice

c. Omelet, soft whole wheat bread Side effects of radiation therapy may include inflammation of the esophagus. Clients should be taught that bland, soft, high-calorie foods are best, along with liquid nutritional supplements. Tomato sauce may prove too spicy for a client with esophagitis. A grilled cheese sandwich is too difficult to swallow with this condition, and orange juice and other foods with citric acid are too caustic.

A client had an acute myocardial infarction. What assessment finding indicates to the nurse that a significant complication has occurred? a. Blood pressure that is 20 mm Hg below baseline b. Oxygen saturation of 94% on room air c. Poor peripheral pulses and cool skin d. Urine output of 1.2 mL/kg/hr for 4 hours

c. Poor peripheral pulses and cool skin Poor peripheral pulses and cool skin may be signs of impending cardiogenic shock and should be reported immediately. A blood pressure drop of 20 mm Hg is not worrisome. An oxygen saturation of 94% is just slightly below normal. A urine output of 1.2 mL/kg/hr for 4 hours is normal.

A nurse is caring for several older clients in the hospital that the nurse identifies as being at high risk for healthcare-associated pneumonia. To reduce this risk, what activity should the nurse delegate to the unlicensed assistive personnel (UAP)? a. Encourage between-meal snacks. b. Monitor temperature every 4 hours. c. Provide oral care every 4 hours. d. Report any new onset of cough.

c. Provide oral care every 4 hours. Oral colonization by gram-negative bacteria is a risk factor for healthcare-associated pneumonia. Good, frequent oral care can help prevent this from developing and is a task that can be delegated to the UAP. Encouraging good nutrition is important, but this will not prevent pneumonia. Monitoring temperature and reporting new cough in clients is important to detect the onset of possible pneumonia but do not prevent it.

A client has an intra-arterial blood pressure monitoring line. The nurse notes bright red blood on the clients sheets. What action should the nurse perform first? a. Assess the insertion site. b. Change the clients sheets. c. Put on a pair of gloves. d. Assess blood pressure.

c. Put on a pair of gloves. For the nurses safety, he or she should put on a pair of gloves to prevent blood exposure. The other actions are appropriate as well, but first the nurse must don a pair of gloves.

A nurse cares for a client with chronic obstructive pulmonary disease (COPD). The client states that he no longer enjoys going out with his friends. How should the nurse respond? a. There are a variety of support groups for people who have COPD. b. I will ask your provider to prescribe you with an antianxiety agent. c. Share any thoughts and feelings that cause you to limit social activities. d. Friends can be a good support system for clients with chronic disorders.

c. Share any thoughts and feelings that cause you to limit social activities. Many clients with moderate to severe COPD become socially isolated because they are embarrassed by frequent coughing and mucus production. They also can experience fatigue, which limits their activities. The nurse needs to encourage the client to verbalize thoughts and feelings so that appropriate interventions can be selected. Joining a support group would not decrease feelings of social isolation if the client does not verbalize feelings. Antianxiety agents will not help the client with social isolation. Encouraging a client to participate in activities without verbalizing concerns also would not be an effective strategy for decreasing social isolation.

A nurse cares for a female client who has a family history of cystic fibrosis. The client asks, Will my children have cystic fibrosis? How should the nurse respond? a. Since many of your family members are carriers, your children will also be carriers of the gene. b. Cystic fibrosis is an autosomal recessive disorder. If you are a carrier, your children will have the disorder. c. Since you have a family history of cystic fibrosis, I would encourage you and your partner to be tested. d. Cystic fibrosis is caused by a protein that controls the movement of chloride. Adjusting your diet will decrease the spread of this disorder.

c. Since you have a family history of cystic fibrosis, I would encourage you and your partner to be tested. Cystic fibrosis is an autosomal recessive disorder in which both gene alleles must be mutated for the disorder to be expressed. The nurse should encourage both the client and partner to be tested for the abnormal gene. The other statements are not true.

A client is receiving an infusion of tissue plasminogen activator (t-PA). The nurse assesses the client to be disoriented to person, place, and time. What action by the nurse is best? a. Assess the clients pupillary responses. b. Request a neurologic consultation. c. Stop the infusion and call the provider. d. Take and document a full set of vital signs.

c. Stop the infusion and call the provider. A change in neurologic status in a client receiving t-PA could indicate intracranial hemorrhage. The nurse should stop the infusion and notify the provider immediately. A full assessment, including pupillary responses and vital signs, occurs next. The nurse may or may not need to call a neurologist.

A nurse cares for a client who has pyelonephritis. The client states, I am embarrassed to talk about my symptoms. How should the nurse respond? a. I am a professional. Your symptoms will be kept in confidence. b. I understand. Elimination is a private topic and shouldnt be discussed. c. Take your time. It is okay to use words that are familiar to you. d. You seem anxious. Would you like a nurse of the same gender to care for you?

c. Take your time. It is okay to use words that are familiar to you. Clients may be uncomfortable discussing issues related to elimination and the genitourinary area. The nurse should encourage the client to use language that is familiar to the client. The nurse should not make promises that cannot be kept, like keeping the clients symptoms confidential. The nurse must assess the client and cannot take the time to stop the discussion or find another nurse to complete the assessment.

A nurse assesses a client with bladder cancer who is recovering from a complete cystectomy with ileal conduit. Which assessment finding should alert the nurse to urgently contact the health care provider? a. The ileostomy is draining blood-tinged urine. b. There is serous sanguineous drainage present on the surgical dressing. c. The ileostomy stoma is pale and cyanotic in appearance. d. Oxygen saturations are 92% on room air.

c. The ileostomy stoma is pale and cyanotic in appearance. A pale or cyanotic stoma indicates impaired circulation to the stoma and must be treated to prevent necrosis. Blood-tinged urine and serous sanguineous drainage are expected after this type of surgery. Oxygen saturation of 92% on room air is at the low limit of normal.

A nurse in a family practice clinic is preparing discharge instructions for a client reporting facial pain that is worse when bending over, tenderness across the cheeks, and postnasal discharge. What instruction will be most helpful? a. Ice packs may help with the facial pain. b. Limit fluids to dry out your sinuses. c. Try warm, moist heat packs on your face. d. We will schedule you for a computed tomography scan this week.

c. Try warm, moist heat packs on your face. This client has rhinosinusitis. Comfort measures for this condition include breathing in warm steam, hot packs, nasal saline irrigations, sleeping with the head elevated, increased fluids, and avoiding cigarette smoke. The client does not need a CT scan.

A student nurse is providing tracheostomy care. What action by the student requires intervention by the instructor? a. Holding the device securely when changing ties b. Suctioning the client first if secretions are present c. Tying a square knot at the back of the neck d. Using half-strength peroxide for cleansing

c. Tying a square knot at the back of the neck To prevent pressure ulcers and for client safety, when ties are used that must be knotted, the knot should be placed at the side of the clients neck, not in back. The other actions are appropriate.

After teaching a client who is recovering from a heart transplant to change positions slowly, the client asks, Why is this important? How should the nurse respond? a. Rapid position changes can create shear and friction forces, which can tear out your internal vascular sutures. b. Your new vascular connections are more sensitive to position changes, leading to increased intravascular pressure and dizziness. c. Your new heart is not connected to the nervous system and is unable to respond to decreases in blood pressure caused by position changes. d. While your heart is recovering, blood flow is diverted away from the brain, increasing the risk for stroke when you stand up.

c. Your new heart is not connected to the nervous system and is unable to respond to decreases in blood pressure caused by position changes. Because the new heart is denervated, the baroreceptor and other mechanisms that compensate for blood pressure drops caused by position changes do not function. This allows orthostatic hypotension to persist in the postoperative period. The other options are false statements and do not correctly address the clients question.

A nurse cares for a client who tests positive for alpha1-antitrypsin (AAT) deficiency. The client asks, What does this mean? How should the nurse respond? a. Your children will be at high risk for the development of chronic obstructive pulmonary disease. b. I will contact a genetic counselor to discuss your condition. c. Your risk for chronic obstructive pulmonary disease is higher, especially if you smoke. d. This is a recessive gene and should have no impact on your health.

c. Your risk for chronic obstructive pulmonary disease is higher, especially if you smoke. The gene for AAT is a recessive gene. Clients with only one allele produce enough AAT to prevent chronic obstructive pulmonary disease (COPD) unless the client smokes. A client with two alleles is at high risk for COPD even if not exposed to smoke or other irritants. The client is a carrier, and children may or may not be at high risk depending on the partners AAT levels. Contacting a genetic counselor may be helpful but does not address the clients current question.

A nurse assesses several clients who have a history of asthma. Which client should the nurse assess first? a. A 66-year-old client with a barrel chest and clubbed fingernails b. A 48-year-old client with an oxygen saturation level of 92% at rest c. A 35-year-old client who has a longer expiratory phase than inspiratory phase d. A 27-year-old client with a heart rate of 120 beats/min

d. A 27-year-old client with a heart rate of 120 beats/min Tachycardia can indicate hypoxemia as the body tries to circulate the oxygen that is available. A barrel chest is not an emergency finding. Likewise, a pulse oximetry level of 92% is not considered an acute finding. The expiratory phase is expected to be longer than the inspiratory phase in someone with airflow limitation.

A nurse assesses a client after administering isosorbide mononitrate (Imdur). The client reports a headache. Which action should the nurse take? a. Initiate oxygen therapy. b. Hold the next dose of Imdur. c. Instruct the client to drink water. d. Administer PRN acetaminophen.

d. Administer PRN acetaminophen. The vasodilating effects of isosorbide mononitrate frequently cause clients to have headaches during the initial period of therapy. Clients should be told about this side effect and encouraged to take the medication with food. Some clients obtain relief with mild analgesics, such as acetaminophen. The clients headache is not related to hypoxia or dehydration; therefore, these interventions would not help. The client needs to take the medication as prescribed to prevent angina; the medication should not be held.

A nurse cares for a client who had a chest tube placed 6 hours ago and refuses to take deep breaths because of the pain. Which action should the nurse take? a. Ambulate the client in the hallway to promote deep breathing. b. Auscultate the clients anterior and posterior lung fields. c. Encourage the client to take shallow breaths to help with the pain. d. Administer pain medication and encourage the client to take deep breaths.

d. Administer pain medication and encourage the client to take deep breaths. A chest tube is placed in the pleural space and may be uncomfortable for a client. The nurse should provide pain medication to minimize discomfort and encourage the client to take deep breaths. The other responses do not address the clients discomfort and need to take deep breaths to prevent complications.

A nurse cares for a client who has kidney stones from secondary hyperoxaluria. Which medication should the nurse anticipate administering? a. Phenazopyridine (Pyridium) b. Propantheline (Pro-Banthine) c. Tolterodine (Detrol LA) d. Allopurinol (Zyloprim)

d. Allopurinol (Zyloprim) Stones caused by secondary hyperoxaluria respond to allopurinol (Zyloprim). Phenazopyridine is given to clients with urinary tract infections. Propantheline is an anticholinergic. Tolterodine is an anticholinergic with smooth muscle relaxant properties.

A nurse assesses clients on the medical-surgical unit. Which client is at greatest risk for bladder cancer? a. A 25-year-old female with a history of sexually transmitted diseases b. A 42-year-old male who has worked in a lumber yard for 10 years c. A 55-year-old female who has had numerous episodes of bacterial cystitis d. An 86-year-old male with a 50pack-year cigarette smoking history

d. An 86-year-old male with a 50pack-year cigarette smoking history The greatest risk factor for bladder cancer is a long history of tobacco use. The other factors would not necessarily contribute to the development of this specific type of cancer.

A nurse cares for a client who is recovering after a nephrostomy tube was placed 6 hours ago. The nurse notes drainage in the tube has decreased from 40 mL/hr to 12 mL over the last hour. Which action should the nurse take? a. Document the finding in the clients record. b. Evaluate the tube as working in the hand-off report. c. Clamp the tube in preparation for removing it. d. Assess the clients abdomen and vital signs.

d. Assess the clients abdomen and vital signs. The nephrostomy tube should continue to have a consistent amount of drainage. If the drainage slows or stops, it may be obstructed. The nurse must notify the provider, but first should carefully assess the clients abdomen for pain and distention and check vital signs so that this information can be reported as well. The other interventions are not appropriate.

A nurse assesses a client who is recovering from a radical nephrectomy for renal cell carcinoma. The nurse notes that the clients blood pressure has decreased from 134/90 to 100/56 mm Hg and urine output is 20 mL for this past hour. Which action should the nurse take? a. Position the client to lay on the surgical incision. b. Measure the specific gravity of the clients urine. c. Administer intravenous pain medications. d. Assess the rate and quality of the clients pulse.

d. Assess the rate and quality of the clients pulse. The nurse should first fully assess the client for signs of volume depletion and shock, and then notify the provider. The radical nature of the surgery and the proximity of the surgery to the adrenal gland put the client at risk for hemorrhage and adrenal insufficiency. Hypotension is a clinical manifestation associated with both hemorrhage and adrenal insufficiency. Hypotension is particularly dangerous for the remaining kidney, which must receive adequate perfusion to function effectively. Re-positioning the client, measuring specific gravity, and administering pain medication would not provide data necessary to make an appropriate clinical decision, nor are they appropriate interventions at this time.

A nurse teaches a client with functional urinary incontinence. Which statement should the nurse include in this clients teaching?a. You must clean around your catheter daily with soap and water. b. Wash the vaginal weights with a 10% bleach solution after each use. c. Operations to repair your bladder are available, and you can consider these. d. Buy slacks with elastic waistbands that are easy to pull down.

d. Buy slacks with elastic waistbands that are easy to pull down. Functional urinary incontinence occurs as the result of problems not related to the clients bladder, such as trouble ambulating or difficulty accessing the toilet. One goal is that the client will be able to manage his or her clothing independently. Elastic waistband slacks that are easy to pull down can help the client get on the toilet in time to void. The other instructions do not relate to functional urinary incontinence.

A nurse teaches a client who is prescribed digoxin (Lanoxin) therapy. Which statement should the nurse include in this clients teaching? a. Avoid taking aspirin or aspirin-containing products. b. Increase your intake of foods that are high in potassium. c. Hold this medication if your pulse rate is below 80 beats/min. d. Do not take this medication within 1 hour of taking an antacid.

d. Do not take this medication within 1 hour of taking an antacid. Gastrointestinal absorption of digoxin is erratic. Many medications, especially antacids, interfere with its absorption. Clients are taught to hold their digoxin for bradycardia; a heart rate of 80 beats/min is too high for this cutoff. Potassium and aspirin have no impact on digoxin absorption, nor do these statements decrease complications of digoxin therapy.

A nurse cares for a client with autosomal dominant polycystic kidney disease (ADPKD). The client asks, Will my children develop this disease? How should the nurse respond? a. No genetic link is known, so your children are not at increased risk. b. Your sons will develop this disease because it has a sex-linked gene. c. Only if both you and your spouse are carriers of this disease. d. Each of your children has a 50% risk of having ADPKD.

d. Each of your children has a 50% risk of having ADPKD. Children whose parent has the autosomal dominant form of PKD have a 50% chance of inheriting the gene that causes the disease. ADPKD is transmitted as an autosomal dominant trait and therefore is not gender specific. Both parents do not need to have this disorder.

A nurse evaluates the following arterial blood gas and vital sign results for a client with chronic obstructive pulmonary disease (COPD): Arterial Blood Gas Results Vital Signs pH = 7.32 PaCO2 = 62 mm HgPaO2 = 46 mm HgHCO3 = 28 mEq/L Heart rate = 110 beats/min Respiratory rate = 12 breaths/min Blood pressure = 145/65 mm Hg Oxygen saturation = 76% Which action should the nurse take first? a. Administer a short-acting beta2 agonist inhaler. b. Document the findings as normal for a client with COPD. c. Teach the client diaphragmatic breathing techniques. d. Initiate oxygenation therapy to increase saturation to 92%.

d. Initiate oxygenation therapy to increase saturation to 92%. Oxygen should be administered to a client who is hypoxic even if the client has COPD and is a carbon dioxide retainer. The other interventions do not address the clients hypoxia, which is the priority.

The nurse is caring for a client who is prescribed a long-acting beta2 agonist. The client states, The medication is too expensive to use every day. I only use my inhaler when I have an attack. How should the nurse respond? a. You are using the inhaler incorrectly. This medication should be taken daily. b. If you decrease environmental stimuli, it will be okay for you to use the inhaler only for asthma attacks. c. Tell me more about your fears related to feelings of breathlessness. d. It is important to use this type of inhaler every day. Lets identify potential community services to help you.

d. It is important to use this type of inhaler every day. Lets identify potential community services to help you. Long-acting beta2 agonists should be used every day to prevent asthma attacks. This medication should not be taken when an attack starts. Asthma medications can be expensive. Telling the client that he or she is using the inhaler incorrectly does not address the clients financial situation, which is the main issue here. Clients with limited incomes should be provided with community resources. Asking the client about fears related to breathlessness does not address the clients immediate concerns.

A nurse cares for a client who has a pleural chest tube. Which action should the nurse take to ensure safe use of this equipment?a. Strip the tubing to minimize clot formation and ensure patency. b. Secure tubing junctions with clamps to prevent accidental disconnections. c. Connect the chest tube to wall suction at the level prescribed by the provider. d. Keep padded clamps at the bedside for use if the drainage system is interrupted.

d. Keep padded clamps at the bedside for use if the drainage system is interrupted. Padded clamps should be kept at the bedside for use if the drainage system becomes dislodged or is interrupted. The nurse should never strip the tubing. Tubing junctions should be taped, not clamped. Wall suction should be set at the level indicated by the devices manufacturer, not the provider.

A nurse is assessing a client who had a myocardial infarction. Upon auscultating heart sounds, the nurse hears the following sound. What action by the nurse is most appropriate? (Click the media button to hear the audio clip.) a. Assess for further chest pain. b. Call the Rapid Response Team. c. Have the client sit upright. d. Listen to the clients lung sounds.

d. Listen to the clients lung sounds. The sound the nurse hears is an S3 heart sound, an abnormal sound that may indicate heart failure. The nurse should next assess the clients lung sounds. Assessing for chest pain is not directly related. There is no indication that the Rapid Response Team is needed. Having the client sit up will not change the heart sound.

An older client with peripheral vascular disease (PVD) is explaining the daily foot care regimen to the family practice clinic nurse. What statement by the client may indicate a barrier to proper foot care? a. I nearly always wear comfy sweatpants and house shoes. b. Im glad I get energy assistance so my house isnt so cold. c. My daughter makes sure I have plenty of lotion for my feet. d. My hands shake when I try to do things requiring coordination.

d. My hands shake when I try to do things requiring coordination. Clients with PVD need to pay special attention to their feet. Toenails need to be kept short and cut straight across. The client whose hands shake may cause injury when trimming toenails. The nurse should refer this client to a podiatrist. Comfy sweatpants and house shoes are generally loose and not restrictive, which is important for clients with PVD. Keeping the house at a comfortable temperature makes it less likely the client will use alternative heat sources, such as heating pads, to stay warm. The client should keep the feet moist and soft with lotion.

A client has the diagnosis of valley fever accompanied by myalgias and arthralgias. What treatment should the nurse educate the client on? a. Intravenous amphotericin B b. Long-term anti- inflammatories c. No specific treatment d. Oral fluconazole (Diflucan)

d. Oral fluconazole (Diflucan) Valley fever, or coccidioidomycosis, is a fungal infection. Many people do not need treatment and the disease resolves on its own. However, the presence of joint and muscle pain indicates a moderate infection that needs treatment with antifungal medications. IV amphotericin is reserved for pregnant women and those with severe infection. Anti-inflammatory medications may be used to treat muscle aches and pain but are not used long term.

A nursing student is caring for a client with an abdominal aneurysm. What action by the student requires the registered nurse to intervene? a. Assesses the client for back pain b. Auscultates over abdominal bruit c. Measures the abdominal girth d. Palpates the abdomen in four quadrants

d. Palpates the abdomen in four quadrants Abdominal aneurysms should never be palpated as this increases the risk of rupture. The registered nurse should intervene when the student attempts to do this. The other actions are appropriate.

A student nurse is assessing the peripheral vascular system of an older adult. What action by the student would cause the faculty member to intervene? a. Assessing blood pressure in both upper extremities b. Auscultating the carotid arteries for any bruits c. Classifying capillary refill of 4 seconds as normal d. Palpating both carotid arteries at the same time

d. Palpating both carotid arteries at the same time The student should not compress both carotid arteries at the same time to avoid brain ischemia. Blood pressure should be taken and compared in both arms. Prolonged capillary refill is considered to be greater than 5 seconds in an older adult, so classifying refill of 4 seconds as normal would not require intervention. Bruits should be auscultated.

The nurse is preparing to change a clients sternal dressing. What action by the nurse is most important? a. Assess vital signs. b. Don a mask and gown. c. Gather needed supplies. d. Perform hand hygiene.

d. Perform hand hygiene. To prevent a sternal wound infection, the nurse washes hands or performs hand hygiene as a priority. Vital signs do not necessarily need to be assessed beforehand. A mask and gown are not needed. The nurse should gather needed supplies, but this is not the priority.

A client undergoing hemodynamic monitoring after a myocardial infarction has a right atrial pressure of 0.5 mm Hg. What action by the nurse is most appropriate? a. Level the transducer at the phlebostatic axis. b. Lay the client in the supine position. c. Prepare to administer diuretics. d. Prepare to administer a fluid bolus.

d. Prepare to administer a fluid bolus. Normal right atrial pressures are from 1 to 8 mm Hg. Lower pressures usually indicate hypovolemia, so the nurse should prepare to administer a fluid bolus. The transducer should remain leveled at the phlebostatic axis. Positioning may or may not influence readings. Diuretics would be contraindicated.

A nurse is caring for a client with acute pericarditis who reports substernal precordial pain that radiates to the left side of the neck. Which nonpharmacologic comfort measure should the nurse implement? a. Apply an ice pack to the clients chest. b. Provide a neck rub, especially on the left side. c. Allow the client to lie in bed with the lights down. d. Sit the client up with a pillow to lean forward on.

d. Sit the client up with a pillow to lean forward on. Pain from acute pericarditis may worsen when the client lays supine. The nurse should position the client in a comfortable position, which usually is upright and leaning slightly forward. Pain is decreased by using gravity to take pressure off the heart muscle. An ice pack and neck rub will not relieve this pain.

A nurse is assessing a client who has a tracheostomy. The nurse notes that the tracheostomy tube is pulsing with the heartbeat as the clients pulse is being taken. No other abnormal findings are noted. What action by the nurse is most appropriate? a. Call the operating room to inform them of a pending emergency case. b. No action is needed at this time; this is a normal finding in some clients. c. Remove the tracheostomy tube; ventilate the client with a bag-valve-mask. d. Stay with the client and have someone else call the provider immediately.

d. Stay with the client and have someone else call the provider immediately. This client may have a tracheainnominate artery fistula, which can be a life-threatening emergency if the artery is breached and the client begins to hemorrhage. Since no bleeding is yet present, the nurse stays with the client and asks someone else to notify the provider. If the client begins hemorrhaging, the nurse removes the tracheostomy and applies pressure at the bleeding site. The client will need to be prepared for surgery.

A client is in the family practice clinic reporting a severe cold that started 4 days ago. On examination, the nurse notes the client also has a severe headache and muscle aches. What action by the nurse is best? a. Educate the client on oseltamivir (Tamiflu). b. Facilitate admission to the hospital. c. Instruct the client to have a flu vaccine. d. Teach the client to sneeze in the upper sleeve.

d. Teach the client to sneeze in the upper sleeve. Sneezing and coughing into ones sleeve helps prevent the spread of upper respiratory infections. The client does have manifestations of the flu (influenza), but it is too late to start antiviral medications; to be effective, they must be started within 24 to 48 hours of symptom onset. The client does not need hospital admission. The client should be instructed to have a flu vaccination, but now that he or she has the flu, vaccination will have to wait until next year.

A client is in the preoperative holding area prior to an emergency coronary artery bypass graft (CABG). The client is yelling at family members and tells the doctor to just get this over with when asked to sign the consent form. What action by the nurse is best? a. Ask the family members to wait in the waiting area. b. Inform the client that this behavior is unacceptable. c. Stay out of the room to decrease the clients stress levels. d. Tell the client that anxiety is common and that you can help.

d. Tell the client that anxiety is common and that you can help. Preoperative fear and anxiety are common prior to cardiac surgery, especially in emergent situations. The client is exhibiting anxiety, and the nurse should reassure the client that fear is common and offer to help. The other actions will not reduce the clients anxiety.

The nurse is reviewing the lipid panel of a male client who has atherosclerosis. Which finding is most concerning? a. Cholesterol: 126 mg/dL b. High-density lipoprotein cholesterol (HDL-C): 48 mg/dL c. Low-density lipoprotein cholesterol (LDL-C): 122 mg/dL d. Triglycerides: 198 mg/dL

d. Triglycerides: 198 mg/dL Triglycerides in men should be below 160 mg/dL. The other values are appropriate for adult males.

A nurse plans care for a client with overflow incontinence. Which intervention should the nurse include in this clients plan of care to assist with elimination? a. Stroke the medial aspect of the thigh. b. Use intermittent catheterization. c. Provide digital anal stimulation. d. Use the Valsalva maneuver.

d. Use the Valsalva maneuver. In clients with overflow incontinence, the voiding reflex arc is not intact. Mechanical pressure, such as that achieved through the Valsalva maneuver (holding the breath and bearing down as if to defecate), can initiate voiding. Stroking the medial aspect of the thigh or providing digital anal stimulation requires the reflex arc to be intact to initiate elimination. Due to the high risk for infection, intermittent catheterization should only be implemented when other interventions are not successful.

A client is being discharged on long-term therapy for tuberculosis (TB). What referral by the nurse is most appropriate? a. Community social worker for Meals on Wheels b. Occupational therapy for job retraining c. Physical therapy for homebound therapy services d. Visiting Nurses for directly observed therapy

d. Visiting Nurses for directly observed therapy Directly observed therapy is often utilized for managing clients with TB in the community. Meals on Wheels, job retraining, and home therapy may or may not be appropriate.

A nurse teaches a client who has a history of heart failure. Which statement should the nurse include in this clients discharge teaching? a. Avoid drinking more than 3 quarts of liquids each day. b. Eat six small meals daily instead of three larger meals. c. When you feel short of breath, take an additional diuretic. d. Weigh yourself daily while wearing the same amount of clothing.

d. Weigh yourself daily while wearing the same amount of clothing. Clients with heart failure are instructed to weigh themselves daily to detect worsening heart failure early, and thus avoid complications. Other signs of worsening heart failure include increasing dyspnea, exercise intolerance, cold symptoms, and nocturia. Fluid overload increases symptoms of heart failure. The client should be taught to eat a heart-healthy diet, balance intake and output to prevent dehydration and overload, and take medications as prescribed. The most important discharge teaching is daily weights as this provides the best data related to fluid retention.

A client has been admitted for suspected inhalation anthrax infection. What question by the nurse is most important? a. Are any family members also ill? b. Have you traveled recently? c. How long have you been ill? d. What is your occupation?

d. What is your occupation? Inhalation anthrax is rare and is an occupational hazard among people who work with animal wool, bone meal, hides, and skin, such as taxidermists and veterinarians. Inhalation anthrax seen in someone without an occupational risk is considered a bioterrorism event and must be reported to authorities immediately. The other questions are appropriate for anyone with an infection.

A nurse cares for a client who has a chest tube. When would this client be at highest risk for developing a pneumothorax? a. When the insertion site becomes red and warm to the touch b. When the tube drainage decreases and becomes sanguineous c. When the client experiences pain at the insertion site d. When the tube becomes disconnected from the drainage system

d. When the tube becomes disconnected from the drainage system Intrathoracic pressures are less than atmospheric pressures; therefore, if the chest tube becomes disconnected from the drainage system, air can be sucked into the pleural space and cause a pneumothorax. A red, warm, and painful insertion site does not increase the clients risk for a pneumothorax. Tube drainage should decrease and become serous as the client heals. Sanguineous drainage is a sign of bleeding but does not increase the clients risk for a pneumothorax.

A nurse cares for a client with end-stage heart failure who is awaiting a transplant. The client appears depressed and states, I know a transplant is my last chance, but I dont want to become a vegetable. How should the nurse respond? a. Would you like to speak with a priest or chaplain? b. I will arrange for a psychiatrist to speak with you. c. Do you want to come off the transplant list? d. Would you like information about advance directives?

d. Would you like information about advance directives? The client is verbalizing a real concern or fear about negative outcomes of the surgery. This anxiety itself can have a negative effect on the outcome of the surgery because of sympathetic stimulation. The best action is to allow the client to verbalize the concern and work toward a positive outcome without making the client feel as though he or she is crazy. The client needs to feel that he or she has some control over the future. The nurse personally provides care to address the clients concerns instead of pushing the clients issues off on a chaplain or psychiatrist. The nurse should not jump to conclusions and suggest taking the client off the transplant list, which is the best treatment option.

A nurse cares for a client who is scheduled for the surgical creation of an ileal conduit. The client states, I am anxious about having an ileal conduit. What is it like to have this drainage tube? How should the nurse respond? a. I will ask the provider to prescribe you an antianxiety medication. b. Would you like to discuss the procedure with your doctor once more? c. I think it would be nice to not have to worry about finding a bathroom. d. Would you like to speak with someone who has an ileal conduit?

d. Would you like to speak with someone who has an ileal conduit? The goal for the client who is scheduled to undergo a procedure such as an ileal conduit is to have a positive self-image and a positive attitude about his or her body. Discussing the procedure candidly with someone who has undergone the same procedure will foster such feelings, especially when the current client has an opportunity to ask questions and voice concerns to someone with first-hand knowledge. Medications for anxiety will not promote a positive self-image and a positive attitude, nor will discussing the procedure once more with the physician or hearing the nurses opinion.

A nurse teaches a client who is recovering from a nephrectomy secondary to kidney trauma. Which statement should the nurse include in this clients teaching? a. Since you only have one kidney, a salt and fluid restriction is required. b. Your therapy will include hemodialysis while you recover. c. Medication will be prescribed to control your high blood pressure. d. You need to avoid participating in contact sports like football.

d. You need to avoid participating in contact sports like football. Clients with one kidney need to avoid contact sports because the kidneys are easily injured. The client will not be required to restrict salt and fluids, end up on dialysis, or have new hypertension because of the nephrectomy.


Related study sets

Microecon - Chapter 9 Concept Check Quiz

View Set

Chapter 8: Power and influence tactics

View Set

Chapter 38: Caring for Clients With Cerebrovascular Disorders

View Set

Trauma, Crisis, Disaster, and Related Disorders Assessment

View Set